You are on page 1of 80

Total Marks : 200

Online Prelims TEST - 29 (TEXTBOOK)


( InsightsIAS Mock Test Series for UPSC Preliminary Exam 2020 ) Mark Scored : 0

1 Consider the following statements with respect to Avian Botulism.


1. Botulinum is a natural toxin produced by a protozoa known as Clostridium botulin.
2. The microorganism causing Botulism is commonly found in the soil, river, and sea water.
3. Botulinum affects both humans and animals.
4. Avian Botulism multiplies and becomes the severest in aerobic and acidic conditions.

Select the correct answer using the codes below.


A. 1 and 4 only
B. 2, 3 and 4 only
C. 2 and 3 only
D. 1, 2, 3 and 4

Correct Answer : C

Answer Justification :

Justification: Statement 1: Botulinum is a natural toxin produced by a bacteria known


as Clostridium botulin. It produces the toxin when it starts reproducing.

S2: The bacteria is commonly found in the soil, river, and sea water. There are around eight types —
A, B, C1, C2, D, E, F, and G — of botulinum toxin and they are distinguishable when diagnosed. But
all types of toxins attack the neurons, which leads to muscle paralysis, states a study.

S3: Botulinum affects both humans and animals but the type of the toxin varies — botulinum C in
birds and A, B and E in humans. The toxin has been recognised as a major cause of mortality in
wild birds since the 1900s.

The avian botulism that caused the mass die-off at Sambhar was caused by the climate, according to
the IVRI report.

Water levels were fluctuating throughout the year. Locals reported that due to a good monsoon this
year, the water level reached the lake bed after a gap of 20 years.

S4: The good monsoon provided a favorable environment for the bacteria to spread. The bacteria
needs anaerobic (absence of oxygen) conditions and does not grow in acidic conditions.

The temperature of the water was about 25 degree Celsius. Its pH ranged between 7.4- 9.84.
It also requires a nutrient-rich substrate, like areas with large amounts of decaying plant or
animal materials. The monsoon brought with it a large population of crustaceans (like
shrimps, crabs, and prawns), invertebrates (snails) and plankton (like algae).
These living organisms are capable of hosting the bacteria for a long period of time.
According to reports, the bacteria is also found in the gills and digestive tracts of healthy fish.
It reproduces through spores and these spores remain dormant for years. They are resistant
to temperature changes and drying. Under favourable conditions, the spores are activated.

Learning: The IVRI report noted that after the monsoon, when the water levels receded, there
might have been an increase in salinity levels which could have led to the death of these living
organisms. At this point in time, the spores could have been activated.

1
Total Marks : 200
Online Prelims TEST - 29 (TEXTBOOK)
( InsightsIAS Mock Test Series for UPSC Preliminary Exam 2020 ) Mark Scored : 0

According to another theory, ‘a bird-to-bird cycle’ could also have led to the tragedy. In such
an event, maggots feeding on dead birds can concentrate the toxin. Birds feeding on dead
birds can get affected.
This was observed in Sambhar too as researchers found only insectivorous and omnivorous
birds affected and not herbivores.
The IVRI report discounted external factors like water pollution and eutrophication (a body of
water becoming overly enriched with minerals and nutrients, in turn inducing excessive
growth of algae) as no farming was being carried out in the vicinity and the runoff from the
same was not possible.

Q Source:
https://www.downtoearth.org.in/news/climate-change/avian-botulism-killed-18-000-birds-at-sambhar
-govt-report-67866

2 The expression “office of profit” has been defined in

A. The Constitution of India


B. The Representation of the People Act, 1951
C. Both (a) and (b)
D. None of the above

Correct Answer : D

Answer Justification :

Justification: The expression “office of profit” has not been defined in the Constitution or in the
Representation of the People Act, 1951. 2. It is for the courts to explain the significance and
meaning of this concept. Over the years, courts have decided this issue in the context of specific
factual situations. But, articles 102 (1) and 191(1) which give effect to the concept of office of profit
prescribe restrictions at the central and state level on lawmakers accepting government positions.

The Supreme Court in Pradyut Bordoloi vs Swapan Roy (2001) outlined the four broad principles for
determining whether an office attracts the constitutional disqualification.

1. First, whether the government exercises control over appointment, removal and performance of
the functions of the office

2. Second, whether the office has any remuneration attached to it

3. Third, whether the body in which the office is held has government powers (releasing money,
allotment of land, granting licenses etc.).

4. Fourth, whether the office enables the holder to influence by way of patronage.

Learning: If an MLA or an MP holds a government office and receives benefits from it, then that
office is termed as an “office of profit”. A person will be disqualified if he holds an office of profit
under the central or state government, other than an office declared not to disqualify its holder by a

2
Total Marks : 200
Online Prelims TEST - 29 (TEXTBOOK)
( InsightsIAS Mock Test Series for UPSC Preliminary Exam 2020 ) Mark Scored : 0

law passed by Parliament or state legislature. Basic disqualification criteria for an MP are laid down
in Article 102 of the Constitution, and for an MLA in Article 191. They can be disqualified for: a)
Holding an office of profit under government of India or state government; b) Being of unsound
mind; c) Being an undischarged insolvent; d) Not being an Indian citizen or for acquiring citizenship
of another country.

Q Source:
https://www.insightsonindia.com/wp-content/uploads/2019/12/Insights-November-2019-Current-Affa
irs-Compilation.pdf

3 “Contract for the Web”, prepared by Sir Tim Berners-Lee, was recently in news. Consider the following
about it.
1. It is a legal document, endorsed by the United Nations.
2. The goal of the contract is to create a standard policy for a Web that benefits all.
3. ‘Protecting people’s privacy and personal data to build online trust’ is a key principle of the
document.

Select the correct answer using the codes below.


A. 1 and 2 only
B. 2 and 3 only
C. 1, 2 and 3
D. 3 only

Correct Answer : B

Answer Justification :

Justification: Sir Tim Berners-Lee, inventor of the World Wide Web, has announced a “Contract for
the Web” — aimed at saving the future of his invention.

What is the Contract for the Web?

The idea is to create a global plan of action for all stakeholders to together commit to building a
“better” Web. The goal is to create a standard policy for a Web that benefits all. The Contract
consists of nine principles — three each for governments, private companies, and individuals and
civil society to endorse. It has been created by representatives from over 80 organisations,
including governments, companies, civil society activists, and academics.

The ‘Contract for the Web’ is not a legal document, or a United Nations document — though
the organisation is in talks with the UN. It cannot currently bend governments or companies — even
those that are on board — to its will.

S2 and S3: Principles in the Contract:

1. Governments will “Ensure everyone can connect to the Internet”, “Keep all of the Internet
available, all of the time”, and “Respect and protect people’s fundamental online privacy and data
rights”.

3
Total Marks : 200
Online Prelims TEST - 29 (TEXTBOOK)
( InsightsIAS Mock Test Series for UPSC Preliminary Exam 2020 ) Mark Scored : 0

2. Companies will “Make the Internet affordable and accessible to everyone”, “Respect and protect
people’s privacy and personal data to build online trust”, and “Develop technologies that support
the best in humanity and challenge the worst”.

3. Citizens will “Be creators and collaborators on the Web”, “Build strong communities that respect
civil discourse and human dignity”, and “Fight for the Web” so that it “remains open and a global
public resource for people everywhere, now and in the future”.

Q Source:
https://www.theguardian.com/technology/2019/nov/24/tim-berners-lee-unveils-global-plan-to-save-th
e-internet

4 According to the Environment Protection Act, 1986, the central government can prohibit or restrict the
location of industries for carrying on certain operations or processes on the basis of considerations like
the
1. Biological diversity of an area
2. Maximum allowable limits of concentration of pollutants for an area
3. Environmentally compatible land use
4. Proximity to protected areas

Select the correct answer using the codes below.


A. 1, 3 and 4 only
B. 2 and 4 only
C. 1 and 3 only
D. 1, 2, 3 and 4

Correct Answer : D

Answer Justification :

Justification: The conditions mentioned above are used to carve out ESZs. The Environment
Protection Act, 1986 does not mention the word “Eco-sensitive Zones”, however:

The section 3(2)(v) of the Act, says that Central Government can restrict areas in which any
industries, operations or processes shall not be carried out or shall be carried out subject to
certain safeguards
Besides the section 5 (1) of this act says that central government can prohibit or restrict the
location of industries and carrying on certain operations or processes on the basis of
considerations like the biological diversity of an area, maximum allowable limits of
concentration of pollutants for an area, environmentally compatible land use, and proximity to
protected areas.

The above two clauses have been effectively used by the government to declare Eco-Sensitive Zones
or Ecologically Fragile Areas (EFA). The same criteria have been used by the government to declare
No Development Zones.

Q Source: ESZ notifications: In news frequently

4
Total Marks : 200
Online Prelims TEST - 29 (TEXTBOOK)
( InsightsIAS Mock Test Series for UPSC Preliminary Exam 2020 ) Mark Scored : 0

5 The Economic Survey 2019-20 introduces the idea of “trust as a public good that gets enhanced with
greater use”. Consider the following about it.
1. It conceptualizes trust as a public good with the characteristics of non-excludability and nonrival
consumption.
2. It states that lack of trust represents an externality where decision makers are not responsible for
some of the consequences of their actions.

Select the correct answer using the codes below.


A. 1 only
B. 2 only
C. Both 1 and 2
D. None of the above

Correct Answer : C

Answer Justification :

Justification: Excerpts from the Survey:

“Trust can be conceptualized as a public good with the characteristics of non-excludability i.e., the
citizens can enjoy its benefits at no explicit financial cost. Trust also has the characteristics of
nonrival consumption i.e., the marginal cost of supplying this public good to an extra citizen is zero.
It is also non-rejectable i.e., collective supply for all citizens means that it cannot be rejected.

Unlike other public goods, trust grows with repeated use and therefore takes time to build. Lack of
trust represents an externality where decision makers are not responsible for some of the
consequences of their actions. Given the importance of trust in an economy, one might reasonably
expect economic theory to address it, especially in the literature on transaction cost economics or
incomplete contracts.

However, this is not the case. Nobel laureate Oliver Williamson who specializes in transaction cost
economics plainly states that there is no such thing as trust within economic activity: ‘It is
redundant at best and can be misleading to use the term “trust” to describe commercial exchange
for which cost-effective safeguards have been devised in support of more efficient exchange.
Calculative trust is a contradiction in terms as per Williamson.

Q Source: Ch 2: Economic Survey 2019-20

6 Consider the following hills/ranges and their locations.


1. Servarayans: Eastern Ghats
2. Agasthyamalai Hills: Western Ghats
3. Anaimalai Hills: Garhjat Range

Select the correct answer using the codes below.


A. 1 and 2 only
B. 2 only
C. 1 and 3 only

5
Total Marks : 200
Online Prelims TEST - 29 (TEXTBOOK)
( InsightsIAS Mock Test Series for UPSC Preliminary Exam 2020 ) Mark Scored : 0

D. 1, 2 and 3

Correct Answer : A

Answer Justification :

Justification: Statement 1: The Servarayans form part of the southern ranges of the Eastern Ghats
System. it also represents the highest peak in southern part of the Eastern Ghats, with the
Solaikaradu peak.

Statement 2: Cardamom hills conjoin the Anaimalai Hills to the northwest, the Palni Hills to the
northeast and the Agasthyamalai Hills to the south as far as the Aryankavu. The crest of the hills
forms the boundary between Kerala and Tamil Nadu. Anamudi in Eravikulam National Park, is the
highest peak in Western Ghats and also the highest point in India south of the Himalayas

Statement 3: They form a southern portion of the Western Ghats. Anamala / Anaimalai Hills are
south of where the Western Ghats are broken by the Palakkad Gap, which in turn is south of the
Nilgiri Hills. They border the state of Kerala on the Southwest and the Cardamom Hills to the
southeast.

Q Source: Additional Research: 11th NCERT: India Physical Geography

7 Vertical equity in taxation implies

A. Taxation is independent of income levels.


B. Similarly situated people pay the same taxes.
C. Better off people pay more taxes.
D. Everyone pays the same amount of tax.

Correct Answer : C

Answer Justification :

Learning: Taxation is based on the idea of Fairness. Though fairness (i.e., the first criteria of a
good tax system) is not always easy to define, economists suggested inclusion of two elements in the
tax system to make it fair namely, horizontal equity and vertical equity.

Individuals in identical or similar situations paying identical or similar taxes is known as horizontal
equity. When ‘better off people pay more taxes it is known as vertical equity. When we combine
both equities, we each closer to the Indian taxation system.

Q Source: Basics of taxation

8 The vision (targets) of the Automotive Mission Plan 2026 includes


1. Growing the sector in value to 5% of India’s GDP by 2026

6
Total Marks : 200
Online Prelims TEST - 29 (TEXTBOOK)
( InsightsIAS Mock Test Series for UPSC Preliminary Exam 2020 ) Mark Scored : 0

2. Bringing the Indian Automotive Industry among the top three of the world in engineering,
manufacture and exports of vehicles & components

Select the correct answer using the codes below.


A. 1 only
B. 2 only
C. Both 1 and 2
D. None of the above

Correct Answer : B

Answer Justification :

Justification: AMP 2026 is the collective vision of the government of India and the automotive
industry on where the various segments of the automotive industry and auto component industry
need to be by 2026 in terms of size and contribution to the overall Indian economy.

The AMP 2026 is aimed at bringing the Indian Automotive Industry among the top three of the
world in engineering, manufacture and exports of vehicles & components; growing in value to over
12% of India GDP and generating an additional 65 million jobs.

Learning: The objective of the Automotive Mission Plan 2026 includes:

1. To propel the Indian Automotive industry to become the engine of the “Make in India”
programme.

2. To make the Indian Automotive Industry a significant contributor to the “Skill India” programme.

3. Promote safe, efficient and comfortable mobility for every person in the country, with an eye on
environmental protection and affordability through both public and personal transport options.

4. To seek increase of net exports of the Indian Automotive industry several fold.

5. Promote comprehensive and stable policy dispensation for all regulations impacting the industry.

Q Source: https://www.insightsonindia.com/2019/12/02/automotive-mission-plan/

9 In Ancient Sanskrit sources, the usage of the words "Yona", "Yauna" or "Yonaka" appears repeatedly,
and particularly in relation to the

A. The southern tip of India that in present day borders the Mannar coast near Sri Lanka
B. Vedic Ritual sacrifices that were supposed to be performed only by the Emperor
C. The art of building esoteric mandalas in places of spiritual importance
D. Greek kingdoms which neighboured and sometimes occupied the Punjab region

Correct Answer : D

7
Total Marks : 200
Online Prelims TEST - 29 (TEXTBOOK)
( InsightsIAS Mock Test Series for UPSC Preliminary Exam 2020 ) Mark Scored : 0

Answer Justification :

Learning: These terms appears repeatedly, and particularly in relation to the Greek kingdoms
which neighboured or sometimes occupied the Punjab region over a period of several centuries
from the 4th century BCE to the first century CE.

Examples are the Seleucid Empire, the Greco-Bactrian Kingdom and the Indo-Greek Kingdom.

The Yavanas are mentioned in detail in Sangam literature epics such as Pattinappalai, describing
their brisk trade with the Early Cholas in the Sangam period.

Q Source: Themes in Indian History – II: 12th NCERT

10 National Health Profile, 2019 highlights substantial health information under major indicators. These
indicators include
1. Demographic indicators, such as population
2. Socio-economic indicators, such as education and employment
3. Health status indicators, such as incidence and prevalence of common communicable and non-
communicable diseases

Select the correct answer using the codes below.


A. 1 only
B. 2 and 3 only
C. 1, 2 and 3
D. 3 only

Correct Answer : C

Answer Justification :

Justification: National Health Profile, 2019 has been released by the Central Bureau of Health
Intelligence (CBHI). This is the 14th edition.

What is NHP? Prepared by the Central Bureau of Health Intelligence (CBHI). Covers comprehensive
information on demographic, socio-economic health status, health finance indicators, health
infrastructure and health of human resources in the country. Objective: To create a versatile
database of health information and making it available to all stakeholders in the healthcare sector.

The NHP highlights substantial health information under major indicators:

1. Demographic indicators (population and vital statistics).

2. Socio-economic indicators (education, employment, housing and amenities, drinking water and
sanitation).

3. Health status indicators (incidence and prevalence of common communicable and non-
communicable diseases and RCH), etc.

8
Total Marks : 200
Online Prelims TEST - 29 (TEXTBOOK)
( InsightsIAS Mock Test Series for UPSC Preliminary Exam 2020 ) Mark Scored : 0

4. The health finance indicators provides an overview of health insurance and expenditure on
health, both public and Out of Pocket Expenditure (OOP), etc.

5. Human resources provides an overview of availability of manpower working in the health sector.

6. Health infrastructure section provides details of medical and dental colleges, AYUSH institutes,
nursing courses and paramedical courses, etc.

Learning: Key findings:

• Life expectancy in India has increased from 49.7 years in 1970-75 to 68.7 years in 2012-16. life
expectancy for females is 70.2 years and 67.4 years for males.

• Diabetes and hypertension rate are high among Indians while dengue and chikungunya are a
cause of great concern to public health.

• Highest population density of 11,320 people per square kilometre was reported by the National
Capital Territory of Delhi (NCT) whereas Arunachal Pradesh reported the lowest population density
of 17.

• High incidence in the young and economically active population. There has been consistent
decrease in the birth rate, death rate and natural growth rate in India since 1991 to 2017.

Q Source:
https://www.insightsonindia.com/wp-content/uploads/2019/12/Insights-November-2019-Current-Affa
irs-Compilation.pdf

11 As per the 2019 World Nuclear Industry Status Report, the nation with the highest number of
operational nuclear reactors is

A. China
B. India
C. Russia
D. USA

Correct Answer : D

Answer Justification :

Learning: The 2019 World Nuclear Industry Status Report provides an overview of the age,
operation, production and construction of nuclear power plants around the world. The report states
that 417 reactor units were operational in 2019 with another 28 in long-term outage. The United
States accounted for the highest number of operational units at 97 and another two are currently
under construction. American reactors have a mean age of 38.9 years and they account for 19.3% of
the country's electricity generation. With 58 reactors, France comes second on the nuclear power
list and its units account for 71.7% of all electricity generation.

9
Total Marks : 200
Online Prelims TEST - 29 (TEXTBOOK)
( InsightsIAS Mock Test Series for UPSC Preliminary Exam 2020 ) Mark Scored : 0

Despite its recent focus on renewables, China has made rapid progress in power plant construction
and it had the world's third-highest total in 2019 at 47. Russia and South Korea came fourth and
fifth on the list with 36 and 23 operational units respectively. Ukraine and Japan are still operating
nuclear power plants despite serious accidents at Chernobyl in 1986 and Fukushima in 2011.
Ukraine still has 15 operational reactors compared to Japan's nine. Japan is notable on the list as
having the highest number of reactors in long-term outage, however, with 24.

Q Source:
https://www.forbes.com/sites/niallmccarthy/2020/02/06/the-countries-operating-the-most-nuclear-re
actors-infographic/#4acbc25b102b

12 Which of the following are Rabi crops?


1. Soybean
2. Linseed
3. Pea
4. Mustard

Choose the correct answer using the codes below:


A. 1 only
B. 2 and 3 only
C. 1, 2, 3 and 4
D. 2, 3 and 4 only

Correct Answer : D

Answer Justification :

Learning: The crops that are sown in the rainy season are called kharif crops. (also known as the
summer or monsoon crop) in India. Kharif crops are usually sown with the beginning of the first
rains in July, during the south-west monsoon season.

The crops that are sown in the winter season are called Rabi crops. (also known as the “winter
crop”) in Pakistan and India. The Rabi means, when the crop is harvested. Crops that are grown in
the winter season, from November to April are called Rabi Crops. Some of the important rabi crops
are wheat, barley, linseed, peas, gram and mustard. Paddy, maize, soyabean, groundnut, cotton,
etc., are kharif crops.

Q Source: http://www.kerenvis.nic.in/Database/Crops_2419.aspx

13 Which of these African nations is closest to Red Sea?

A. Sudan
B. Mali
C. Uganda

10
Total Marks : 200
Online Prelims TEST - 29 (TEXTBOOK)
( InsightsIAS Mock Test Series for UPSC Preliminary Exam 2020 ) Mark Scored : 0

D. Ghana

Correct Answer : A

Answer Justification :

Learning: Red sea forms the coast of Sudan.

Q Source: Map based questions Africa

14 Uighurs, recently seen in news, are a minority community concentrated in

A. Iran
B. Myanmar
C. Turkey
D. China

Correct Answer : D

Answer Justification :

Justification: Uighurs are a Muslim minority community concentrated in the country’s


northwestern Xinjiang province. They claim closer ethnic ties to Turkey and other central Asian
countries than to China, by brute — and brutal — force.

Why is China targeting the Uighurs?

11
Total Marks : 200
Online Prelims TEST - 29 (TEXTBOOK)
( InsightsIAS Mock Test Series for UPSC Preliminary Exam 2020 ) Mark Scored : 0

Xinjiang is technically an autonomous region within China — its largest region, rich in minerals, and
sharing borders with eight countries, including India, Pakistan, Russia and Afghanistan.

Over the past few decades, as economic prosperity has come to Xinjiang, it has brought with it in
large numbers the majority Han Chinese, who have cornered the better jobs, and left the Uighurs
feeling their livelihoods and identity were under threat.

This led to sporadic violence, in 2009 culminating in a riot that killed 200 people, mostly Han
Chinese, in the region’s capital Urumqi. And many other violent incidents have taken place since
then.

Beijing also says Uighur groups want to establish an independent state and, because of the Uighurs’
cultural ties to their neighbours, leaders fear that elements in places like Pakistan may back a
separatist movement in Xinjiang.

Therefore, the Chinese policy seems to have been one of treating the entire community as suspect,
and launching a systematic project to chip away at every marker of a distinct Uighur identity.

Q Source: In news sometimes

15 Consider the following statements.


1. Supplementary demands for grants find mention in the Constitution of India.
2. Supplementary demands for grants need to be placed for approval only in Lok Sabha.
3. The demands for supplementary grants are made after the expenditure has actually been incurred
and after the financial year to which it relates, has expired.

Select the correct answer using the codes below.


A. 1 and 2 only
B. 1 only
C. 2 and 3 only
D. 1, 2 and 3

Correct Answer : B

Answer Justification :

Justification: Statement 1 and 2: Supplementary grants are the additional grant required to meet
the required expenditure of the government. They are to be presented in both the houses.

Supplementary, additional or excess grants and Votes on account, votes of credit and
exceptional grants are mentioned in the Constitution of India 1949.
Article 115: Supplementary, additional or excess grants.
Article 116: Votes on account, votes of credit and exceptional grants.

Statement 3: When grants, authorised by the Parliament, fall short of the required expenditure, an
estimate is presented before the Parliament for Supplementary or Additional grants. These grants
are presented and passed by the Parliament before the end of the financial year.

12
Total Marks : 200
Online Prelims TEST - 29 (TEXTBOOK)
( InsightsIAS Mock Test Series for UPSC Preliminary Exam 2020 ) Mark Scored : 0

The Demand for Excess Grants (not supplementary grants) is made after the actual expenditure is
incurred and is presented to the Parliament after the end of the financial year in which the expenses
were made.

Learning: When actual expenditure incurred exceeds the approved grants of the Parliament, the
Ministry of Finance and Ministry of Railways presents a Demand for Excess Grant. The Comptroller
and Auditor General of India bring such excesses to the notice of the Parliament. The Public
Accounts Committee examines these excesses and gives recommendations to the Parliament.

Q Source: Chapter on Parliament: IPML

16 Consider the following statements.


1. Unlike in command economies where prices are determined by the government, in a market
economy, price of a good is determined by the interaction of the forces of supply and demand.
2. In a market economy, decisions about what and how much to produce is not centrally made,
whereas in a command economy, such decisions may be made centrally.

Select the correct answer using the codes below.


A. 1 only
B. 2 only
C. Both 1 and 2
D. None of the above

Correct Answer : C

Answer Justification :

Justification and Learning: Market economies and command economies occupy two polar
extremes in the organization of economic activity. The primary differences lie in the division of
labor, or factors of production, and the mechanisms that determine prices. The activity in a market
economy is unplanned; it is not organized by any central authority but is determined by the supply
and demand of goods and services. The United States, England, and Japan are all examples of
market economies.

Alternatively, a command economy is organized by a centralized government which owns most, if


not all, businesses and whose officials direct all the factors of production. China, North Korea, and
the former Soviet Union are all examples of command economies. In reality, all economies blend
some combination of market and command economies.

Consumer preferences and resource scarcity determine which goods are produced and in
what quantity; the prices in a market economy act as signals to producers and consumers
who use these price signals to help make decisions. Governments play a minor role in the direction
of economic activity.

Since decision-making is centralized in a command economy, the government controls all


of the supply and sets all of the demand. Prices cannot arise naturally like in a market
economy, so prices in the economy must be set by government officials.

13
Total Marks : 200
Online Prelims TEST - 29 (TEXTBOOK)
( InsightsIAS Mock Test Series for UPSC Preliminary Exam 2020 ) Mark Scored : 0

In a command economy, macroeconomic and political considerations determine resource allocation,


whereas, in a market economy, the profits and losses of individuals and firms determine resource
allocation. Command economies are concerned with providing basic necessities and opportunities to
all members.

Text cited from: Investopedia


(https://www.investopedia.com/ask/answers/100314/whats-difference-between-market-economy-and
-command-economy.asp)

Q Source: Ch 2: Economic Survey 2019-20

17 Consider the following statements.


1. Transparency International publishes the Index of Politically Connected (IPC) firms.
2. Democracy Watch and Transparency International jointly publish Corruption Perception Index.

Select the correct answer using the codes below.


A. 1 only
B. 2 only
C. Both 1 and 2
D. None of the above

Correct Answer : D

Answer Justification :

Justification and Learning: Statement 1: The incentive for corporates to become politically
connected has been recognized by economists for quite some time. It can take the form of lighter
taxation, preferential treatment/procurement, relaxed regulatory oversight etc.

The Index of Politically connected companies (IPC) was analysed by a Mumbai-based firm Ambit
Capital. As the Economic survey 2019-20 shows in Chapter 4, prior to 2011, cronyism paid a firm
and its shareholders. The index of “connected” firms - as defined and constructed by Ambit Capital
consistently outperformed the BSE 500 index.

This rise in IPC is driven by the presumption that connected companies will gain from the decisions
of the new government. Some analysts say this is a misplaced expectation.

Read more here


https://www.livemint.com/Home-Page/HI9tpqtGbzoqebjWxIsy3L/Politically-connected-firms-lost-mor
e-than-market.html and
https://qz.com/india/223351/politically-connected-companies-are-making-a-comeback-on-indian-mar
kets/

Statement 2: The corruption perception index, which Transparency International tracks across
countries, shows India at its lowest point in recent years in 2011. Since 2013, India has improved
significantly on this index. The phenomenon of trust deficit that developed in India during this
period is also reflected in many other measures.

14
Total Marks : 200
Online Prelims TEST - 29 (TEXTBOOK)
( InsightsIAS Mock Test Series for UPSC Preliminary Exam 2020 ) Mark Scored : 0

We have covered this index in previous tests.

Q Source: Ch 2: Economic Survey 2019-20

18 Who among the following translated Patanjali’s work on Sanskrit grammar into Arabic?

A. Amir Khusrau
B. Al-Idrisi
C. Ibn Abi Ishaq
D. Al-Biruni

Correct Answer : D

Answer Justification :

Learning: Muslim astronomer, mathematician, ethnographist, anthropologist, historian, and


geographer. Al-Biruni lived during a period of unusual political turmoil in the eastern Islamic world.
He served more than six different princes.

Al-Biruni spent years in the company of Brahmana priests and scholars, learning Sanskrit, and
studying religious and philosophical texts.

Al-Biruni’s expertise in several languages allowed him to compare languages and translate texts.

He translated several Sanskrit works, including Patanjali’s work on grammar, into Arabic. For his
Brahmana friends, he translated the works of Euclid (a Greek mathematician) into Sanskrit.

Q Source: Page 116: Themes in Indian History – II: 12th NCERT

19 As per Arthashastra, good governance is based on which of the following branches of knowledge?
1. Jnana (knowledge through enquiry)
2. Varta (economic policy)
3. Dandaneeti (law and enforcement)
4. Anvikshiki (philosophical and ethical framework)
5. Trayi (cultural context)

Select the correct answer using the codes below.


A. 1, 2, 3 and 4 only
B. 2, 3 and 4 only
C. 1 and 5 only
D. 2, 3, 4 and 5 only

Correct Answer : D

15
Total Marks : 200
Online Prelims TEST - 29 (TEXTBOOK)
( InsightsIAS Mock Test Series for UPSC Preliminary Exam 2020 ) Mark Scored : 0

Answer Justification :

Justification: References of Arthashastra are scattered in the Survey, and can be useful for Ethics
paper and for students of Public administration.

Arthashastra explicitly presents its intellectual framework right in the beginning by stating that
good governance is based on the following branches of knowledge: Varta (economic policy),
Dandaneeti (law and enforcement), Anvikshiki (philosophical and ethical framework) and Trayi
(cultural context).

The importance of Anvikshiki in Kautilya’s writings is often ignored but is critical to understanding
his worldview. Interestingly this mirrors Adam Smith who did not just advocate the “invisible hand”
but equally the importance of “mutual sympathy” (i.e. trust).

The same idea is reflected in the writings of Friedrich Hayek, who advocated not only economic
freedom but also a set of general rules and social norms that applies evenly to everyone.

Q Source: Ch 2: Economic Survey 2019-20

20 Consider the following statements.


1. Glucose is on the World Health Organization's List of Essential Medicines.
2. Glucose is stored in plants as starch and in animals as glycogen.
3. Bacteria can derive energy from Glucose.

Select the correct answer using the codes below.


A. 1 and 2 only
B. 2 only
C. 1 and 3 only
D. 1, 2 and 3

Correct Answer : D

Answer Justification :

Justification: Statement 1 and 3: Glucose is a ubiquitous fuel in biology. It is used as an energy


source in most organisms, from bacteria to humans, through either aerobic respiration, anaerobic
respiration, or fermentation. Glucose is the human body's key source of energy, through aerobic
respiration, providing about 3.75 kilocalories (16 kilojoules) of food energy per gram.

Glucose is on the World Health Organization's List of Essential Medicines, the most important
medications needed in a basic health system.

Statement 2: Glucose is a simple sugar with the molecular formula C6H12O6, which means that it
is a molecule that is made of six carbon atoms, twelve hydrogen atoms, and six oxygen atoms.
Glucose circulates in the blood of animals as blood sugar.

It is made during photosynthesis from water and carbon dioxide, using energy from sunlight. It is
the most important source of energy for cellular respiration. Glucose is stored as a polymer, in
16
Total Marks : 200
Online Prelims TEST - 29 (TEXTBOOK)
( InsightsIAS Mock Test Series for UPSC Preliminary Exam 2020 ) Mark Scored : 0

plants as starch and in animals as glycogen.

Q Source: based on past year UPSC papers

21 With respect to Socioeconomic High-resolution Rural-Urban Geographic Dataset on India (SHRUG),


consider the following statements.
1. It contains variables that describe the demographic, socioeconomic, firm and political infrastructure
of every district in India for the last few decades.
2. It is published by the World Bank (WB) based on the reports of UNICEF and United Nations
Development Program (UNDP).

Select the correct answer using the codes below.


A. 1 only
B. 2 only
C. Both 1 and 2
D. None of the above

Correct Answer : A

Answer Justification :

Justification: Economic Survey 2019-20 presents entrepreneurship data relating to the physical
and social infrastructure of a district which was partly accumulated from the Socioeconomic High-
resolution Rural-Urban Geographic Dataset on India (SHRUG) which comprises of a set of variables
that describe the extent of socio-economic development in India.

Among other things, SHRUG contains variables that describe the demographic, socioeconomic, firm
and political infrastructure of every district between 1990–2018 that it cumulates from a variety of
data sources.

These sources comprise data from the Census of India relating to the years 1991, 2001 and 2011,
Socio-Economic Caste Census of 2012, and Pradhan Mantri Gram Sadak Yojana (PMGSY) public
data to construct a variety of indices that describe the social and physical infrastructure of every
district in India.

SHRUG is available at http://www.devdatalab.org/shrug_download/

Q Source: Ch 2: Economic Survey 2019-20

22 Consider the following statements.


1. ‘Sensex’ is a market-weighted stock market index of 30 well-established and financially sound
companies listed on the Bombay Stock Exchange (BSE).
2. The number of Sensex firms in services sector has reduced while those in manufacturing sector has
increased in the last three decades.
3. DOLLEX-30, launched by BSE, is a dollar-linked version of the SENSEX.

17
Total Marks : 200
Online Prelims TEST - 29 (TEXTBOOK)
( InsightsIAS Mock Test Series for UPSC Preliminary Exam 2020 ) Mark Scored : 0

Select the correct answer using the codes below.


A. 1 only
B. 2 and 3 only
C. 1 and 3 only
D. 1, 2 and 3

Correct Answer : C

Answer Justification :

Justification: The BSE SENSEX (also known as the S&P Bombay Stock Exchange or Sensitive
Index or simply the SENSEX) is a free-float market-weighted stock market index of 30 well-
established and financially sound companies listed on Bombay Stock Exchange.

A capitalization-weighted (or "cap-weighted") index, also called a market-value-weighted index is a


stock market index whose components are weighted according to the total market value of their
outstanding shares. Every day an individual stock's price changes and thereby changes a stock
index's value.

The 30 component companies which are some of the largest and most actively traded stocks, are
representative of various industrial sectors of the Indian economy.

Published since 1 January 1986, the S&P BSE SENSEX is regarded as the pulse of the domestic
stock markets in India. The base value of the SENSEX was taken as 100 on 1 April 1979 and its base
year as 1978–79. In 2001 BSE launched DOLLEX-30, a dollar-linked version of the SENSEX.

18
Total Marks : 200
Online Prelims TEST - 29 (TEXTBOOK)
( InsightsIAS Mock Test Series for UPSC Preliminary Exam 2020 ) Mark Scored : 0

Statement 2: India has followed an idiosyncratic growth pattern, wherein the prime mover of the
economy has shifted from agriculture to services. Almost 60 per cent of Indian GDP is attributable
to the services sector. As shown by Figure 6, the number of Sensex firms in manufacturing has
reduced while those in services has increased between 1988 and 2019. Thus, over the years, the
share of services sector in the total number of companies on the Sensex has changed from being
negligible in the 1980s to the dominant status today.

Q Source: AR: Ch 3: Economic Survey 2019-20

23 Which of the following is correct with regard to the role and functions of National Disaster Response
Force (NDRF)?
1. It imparts basic and operational level training to State Response Forces like Police and Home
guards.
2. It is responsible for organizing Public Awareness Campaigns about disaster management.
3. It is the statutorily sanctioned force for guarding sensitive international borders during a disaster.

Select the correct answer using the codes below.


A. 1 and 2 only
B. 2 and 3 only
C. 1 only
D. None of the above

Correct Answer : A

Answer Justification :

Justification: Others include:

Specialized response during disasters.


Proactive deployment during impending disaster situations.
Acquire and continually upgrade its own training and skills.
Liaison, Reconnaissance, Rehearsals and Mock Drills.
Impart basic and operational level training to State Response Forces (Police, Civil Defence
and Home Guards).
Community Capacity Building Programme.
Organize Public Awareness Campaigns.

Q Source: As mentioned above

24 In the Gupta administration, Uparikas were responsible for

A. Governing a province
B. Looking after charities and trusts
C. Managing royal exchequer
D. Administering foreign affairs

19
Total Marks : 200
Online Prelims TEST - 29 (TEXTBOOK)
( InsightsIAS Mock Test Series for UPSC Preliminary Exam 2020 ) Mark Scored : 0

Correct Answer : A

Answer Justification :

Learning: Provinces in the Gupta Empire were known as Bhuktis and provincial governors as
Uparikas. They were mostly chosen from among the princes.

Other officials:

A high official called was mentioned in the Gupta inscriptions known as Sandivigraha, most
probably minister for foreign affairs.

The king was assisted in his administration by a council consisting of a chief minister, a Senapati or
commanderin- chief of the army and other important officials.

The king maintained a close contact with the provincial administration through a class of officials
called Kumaramatyas and Ayuktas.

Q Source: Revision: Chapter 9: 11th Std. TamilNadu History Textbook

25 Starlink network project concerns

A. Asteroid detection
B. Space internet
C. Freight services in North-South International Corridor
D. Networking of Ivy League institutions

Correct Answer : B

Answer Justification :

Justification: Currently, about 4 billion people, more than half the world’s population, do not have
access to reliable Internet networks. And that is because the traditional ways to deliver the Internet
— fibre-optic cables or wireless networks — cannot take it everywhere on Earth. In many remote
areas, or places with difficult terrain, it is not feasible or viable to set up cables or mobile towers.

SpaceX, the world’s leading private company in space technology, has launched 60 satellites into
the low earth orbit, under the Starlink network project.

The Starlink network is one of several ongoing efforts to start beaming data signals from space.
Under the project, the company intends to evolve into a constellation of nearly 12,000 satellites. The
aim is to provide low-cost and reliable space-based Internet services to the world. The project
announced in 2015, has now 122 satellites in the orbit.

The project ensures that reliable and uninterrupted Internet services are universally available in
every part of the globe.

20
Total Marks : 200
Online Prelims TEST - 29 (TEXTBOOK)
( InsightsIAS Mock Test Series for UPSC Preliminary Exam 2020 ) Mark Scored : 0

Q Source: In news

26 Carbohydrate is a bio-molecule consisting of various elements. Which of the following is NOT a major
constituent of carbohydrates?

A. Carbon
B. Hydrogen
C. Nitrogen
D. Oxygen

Correct Answer : C

Answer Justification :

Justification: Option C: Carbohydrates are made of carbon (C), hydrogen (H) and oxygen (O)
atoms, usually with a hydrogen–oxygen atom ratio of 2:1 (as in water); in other words, with the
empirical formula Cm(H2O)n. There may be little or no Nitrogen.

Learning: Carbohydrates perform numerous roles in living organisms. Polysaccharides serve for
the storage of energy (e.g. starch and glycogen) and as structural components (e.g. cellulose in
plants and chitin in arthropods).

For e.g. the 5-carbon monosaccharide ribose is an important component of coenzymes (e.g.
ATP, FAD and NAD) and the backbone of the genetic molecule known as RNA. The related
deoxyribose is a component of DNA.
Saccharides and their derivatives include many other important biomolecules that play key
roles in the immune system, fertilization, preventing pathogenesis, blood clotting, and
development.
Carbohydrates are found in a wide variety of foods. The important sources are cereals,
potatoes, sugarcane, etc. Starch and sugar are the important carbohydrates in our diet.

Q Source: Based on past year UPSC papers

27 Natura 2000 is

A. the largest coordinated network of protected areas in the world


B. a GHG emission standard of European Union (EU) recently admitted by the Board of
Emissions in UNEP
C. a list of most vulnerable species as compiled by the IUCN
D. a non-profit organization that helps conserve small islands in LDCs

Correct Answer : A

Answer Justification :

21
Total Marks : 200
Online Prelims TEST - 29 (TEXTBOOK)
( InsightsIAS Mock Test Series for UPSC Preliminary Exam 2020 ) Mark Scored : 0

Learning: Natura 2000 is a network of core breeding and resting sites for rare and threatened
species, and some rare natural habitat types which are protected in their own right.

It stretches across all 28 EU countries, both on land and at sea.

The aim of the network is to ensure the long-term survival of Europe's most valuable and threatened
species and habitats, listed under both the Birds Directive and the Habitats Directive.

Natura 2000 is not a system of strict nature reserves from which all human activities would be
excluded. While it includes strictly protected nature reserves, most of the land remains privately
owned.

The approach to conservation and sustainable use of the Natura 2000 areas is much wider, largely
centered on people working with nature rather than against it.

Q Source: http://ec.europa.eu/environment/nature/natura2000/index_en.htm

28 The 2019 Brown to Green Report was published by the Climate Transparency partnership, an
international research collaboration. The report is the most comprehensive review of the climate
performance of

A. SAARC countries
B. OECD countries
C. G20 countries
D. Asia-Pacific economies

Correct Answer : C

Answer Justification :

Justification: The report maps their achievements and drawbacks in their efforts to reduce
emissions, adapt to climate impacts and green the financial system.

Key findings of the report:

1. Carbon emissions from the world's 20 biggest economies, including India, are rising.

2. None of the G20 countries have plans that will help them achieve the target. Many of the current
2030 climate targets under the Paris Agreement (Nationally Determined Contributions or NDCs)
are too weak, with about half of the G20 countries projected to meet or overachieve their
inadequate NDCs.

3. Energy-related carbon dioxide emissions in G20 countries shot up by 1.8 per cent in 2018 due to
rising energy demand.

4. Energy supply is not getting cleaner: despite a more than five per cent rise in G20 total

22
Total Marks : 200
Online Prelims TEST - 29 (TEXTBOOK)
( InsightsIAS Mock Test Series for UPSC Preliminary Exam 2020 ) Mark Scored : 0

renewable energy supply in 2018, the share of fossil fuels in the G20 energy mix remains at 82 per
cent.

5. While renewables now account for 25.5 per cent of power generation, this is not sufficient to
outweigh the growth of emissions from fossil fuel sources.

6. Low-carbon fuels need to increase roughly 10 times by 2050 to keep global warming below 1.5
degrees Celsius.

7. G20 emissions in the building sector grew more than in any other sector in 2018 (4.1 per cent).
Retrofitting existing buildings challenges all G20 and especially OECD countries. New buildings
have to be near zero-energy by 2020-25 to keep global warming below 1.5 degrees.

Q Source: https://www.climate-transparency.org/g20-climate-performance/g20report2019

29 Consider the following statements about the formation of the state government after elections.
1. The Governor can only summon the new House for the first sitting only after a new government is
sworn in and the Cabinet has suggested a suitable date.
2. After a party formation is sworn in, the Constitution mandates that it needs to pass a floor test to
ascertain confidence in the legislature.

Select the correct answer using the codes below.


A. 1 only
B. 2 only
C. Both 1 and 2
D. None of the above

Correct Answer : C

Answer Justification :

Background: The Governor would be expected to go as per an order of preference set out in the
Sarkaria Commission recommendations, which have also been ratified by the Supreme Court.

By the order of preference, the Governor can invite:

1. A pre-poll alliance of parties.


2. Invite the single largest party which stakes a claim to form government.
3. Invite a post-poll alliance of parties, with all the partner in the coalition joining the
government.
4. Invite a post-poll alliances of parties, with some becoming part of the government and some
supporting from outside.

Justification: S1: The Governor can only summon the new House for the first sitting only after a
new government is sworn in and the Cabinet has suggested a suitable date. The process of
swearing-in of the newly elected members and appointment of the new Speaker can be held
thereafter.

23
Total Marks : 200
Online Prelims TEST - 29 (TEXTBOOK)
( InsightsIAS Mock Test Series for UPSC Preliminary Exam 2020 ) Mark Scored : 0

S2: Once any formation is sworn in, it will need to pass the floor test, which will reveal whether the
executive enjoys the confidence of the legislature as mandated by the Constitution. In the floor test,
the person sworn in as the CM has to prove that s/he enjoys the confidence of the House. If the
confidence motion fails, the Chief Minister has to resign. If more than one person stake claim to
form the government and the majority is not clear, the Governor has the powers to call a special
session to assess who has the majority.

Q Source: Basics: Polity: IPML

30 What is/are the relative advantages of signalling communication beams from low earth orbits (LEO)
compared to geostationary orbits (GEO)?
1. Signals from LEO have a lower latency than GEO.
2. Owing to their lower height, LEO signals cover a relatively larger area than the GEOs.

Select the correct answer using the codes below.


A. 1 only
B. 2 only
C. Both 1 and 2
D. None of the above

Correct Answer : A

Answer Justification :

Justification: Signals from geostationary orbit can cover a very large part of the Earth. Signals
from one satellite can cover roughly a third of the planet — and three to four satellites would be
enough to cover the entire Earth.

Also, because they appear to be stationary, it is easier to link to them. Then what’s the issue? There
is a time lag — called latency — between a user seeking data, and the server sending that data. And
because data transfers cannot happen faster than the speed of light (in reality, they take place at
significantly lower speeds), the longer the distance that needs to be covered the greater is the time
lag, or latency.

A transmission from a satellite in geostationary orbit has a latency of about 600 milliseconds. A
satellite in the lower orbit, 200-2,000 km from the Earth’s surface, can bring the lag down to
20-30 milliseconds, roughly the time it takes for terrestrial systems to transfer data.

Concerns over LEOs: Owing to their lower height, their signals cover a relatively small area. As a
result, many more satellites are needed in order to reach signals to every part of the planet. Other
issues include: Increased space debris, increased risk of collisions, and the concern of astronomers
that these constellations of space Internet satellites will make it difficult to observe other space
objects, and to detect their signals.

Q Source: Basics of telecommunication: SpaceX plans for massive space internet services

24
Total Marks : 200
Online Prelims TEST - 29 (TEXTBOOK)
( InsightsIAS Mock Test Series for UPSC Preliminary Exam 2020 ) Mark Scored : 0

31 The first multilateral bloc to declare a ‘climate emergency’ is

A. European Union
B. MERCOSUR
C. Commonwealth Nations
D. Asia–Pacific Economic Organization (APEC)

Correct Answer : A

Answer Justification :

Justification: What is Climate Emergency? There is no single definition of what that means but
many local areas say they want to be carbon-neutral by 2030. It varies. For example, for UK
government it is to reduce carbon emissions by 80% (compared to 1990 levels) by 2050.

European Union (EU) has become the first multilateral bloc to declare climate emergency. A
resolution in this regard was recently passed.

Implications: The symbolic move is expected to pressurise countries to act ahead of the United
Nations summit on climate change that starts on December 2 in Spain. The resolution will have
more symbolic effect than practical impact, and is designed to pressure EU governments to adopt a
commitment for all of the EU to reduce emissions to net zero.

Who else have declared climate emergency? Similar climate emergency declarations have already
been made in several EU member states, including Spain, France and the United Kingdom. Outside
Europe only Canada, Argentina and Bangladesh have declared a climate emergency.

Q Source: Recently in news

32 Consider the following statements.


1. An Unstarred Question is one to which written answer is desired by the member and is deemed to
be laid on the Table of the House by Minister.
2. A Starred question is not called for an oral answer in the House and no supplementary question can
be asked thereon.
3. No questions can be addressed directly to a Private Member of the Parliament, unless the question
relates to a Point of Order or Privileges of the member inside the house.

Select the correct answer using the codes below.


A. 1 only
B. 2 and 3 only
C. 1, 2 and 3
D. 1 and 2 only

Correct Answer : A

Answer Justification :
25
Total Marks : 200
Online Prelims TEST - 29 (TEXTBOOK)
( InsightsIAS Mock Test Series for UPSC Preliminary Exam 2020 ) Mark Scored : 0

Justification: Statement 1: Members have a right to ask questions to elicit information on matters
of public importance within the special cognizance of the Ministers concerned. The questions are of
four types:

Type 1: Unstarred Questions: An Unstarred Question is one to which written answer is


desired by the member and is deemed to be laid on the Table of the House by Minister. Thus
it is not called for oral answer in the House and no supplementary question can be asked
thereon.
Type 2: Starred Questions: A Starred Question is one to which a member desires an oral
answer from the Minister in the House and is required to be distinguished by him/her with an
asterisk. Answer to such a question may be followed by supplementary questions by members

Statement 3: Type 3: Questions to Private Members: A Question may also be addressed to a Private
Member (Under Rule 40 of the Rules of Procedure and Conduct of Business in Lok Sabha), provided
that the subject matter of the question relates to some Bill, Resolution or other matter connected
with the business of the House for which that Member is responsible. The procedure in regard to
such questions is same as that followed in the case of questions addressed to a Minister with such
variations as the Speaker may consider necessary.

Learning: Type 4: Short Notice Questions: A member may give a notice of question on a matter of
public importance and of urgent character for oral answer at a notice less than 10 days prescribed
as the minimum period of notice for asking a question in ordinary course. Such a question is known
as ‘Short Notice Question’.

Q Source:
https://www.insightsonindia.com/wp-content/uploads/2019/12/Insights-November-2019-Current-Affa
irs-Compilation.pdf

33 Consider the following statements about Neutrinos.


1. They can be weaponized.
2. Human body cannot tolerate exposure to Neutrinos.
3. They are completely massless.

Choose the correct answer using the codes below.


A. 1 and 2 only
B. 2 only
C. 1 and 3 only
D. None of the above

Correct Answer : D

Answer Justification :

Justification: Neutrinos are the information bearers of the universe — which are almost never lost
in their path. India’s effort in studying neutrinos at INO may help us unravel the deepest mystery of
the universe — why there is more matter than antimatter in the universe.

26
Total Marks : 200
Online Prelims TEST - 29 (TEXTBOOK)
( InsightsIAS Mock Test Series for UPSC Preliminary Exam 2020 ) Mark Scored : 0

Neutrinos are the least harmful of all elementary particles, as they almost never react with
solid bodies.
Also, people tend to confuse the “neutrino” for the “neutron”. This has also led to the
confusion that neutrinos can be weaponised, which is far from the truth. They are not used to
trigger fission reactions, but are instead produced as a result of the reactions.
Neutrinos have mass and as shown recently, they change their masses too.
Neutrinos, named as such because they are electrically neutral, are leptons, and so are not
affected by the strong force either.

Q Source: Frequently in news

34 Consider the following about the Bharhut Stupa.


1. The stupa was first built by Ashoka.
2. Numerous birth stories of the Buddha have been depicted on its railings.
3. It also bears an inscription of Queen Maya’s dream.

Select the correct answer using the codes below.


A. 1 and 2 only
B. 2 and 3 only
C. 1 and 3 only
D. 1, 2 and 3

Correct Answer : D

Answer Justification :

Justification: Statement 1: The Bharhut stupa (in MP) was first built by the Maurya king Ashoka in
the 3rd century BCE, and many works of art were added during the Shunga period, with many
friezes from the 2nd century BCE.

Statement 2: They contain numerous birth stories of the Buddha's previous lives, or Jataka tales.
Many of them are in the shape of large, round medallions.

Statement 3: Queen Maya’s dream, preceding the birth of the Buddha, is also a major theme on the
railing of the Bharhut “stupa”.

Q Source: Chapter 4: 11th NCERT: An Introduction to Indian Arts

35 Delimitation of constituencies and allocation of seats in the Parliament and state legislatures are
dealt by

A. Representation of Peoples Act 1950 (RPA Act 1950)


B. Only the constitution of India
C. Rules Made by the Presiding officer of Each house

27
Total Marks : 200
Online Prelims TEST - 29 (TEXTBOOK)
( InsightsIAS Mock Test Series for UPSC Preliminary Exam 2020 ) Mark Scored : 0

D. Orders issued by the Secretary-General of the corresponding houses

Correct Answer : A

Answer Justification :

Learning: The RPA provides for the following:

Qualification of voters.
Preparation of electoral rolls.
Delimitation of constituencies.
Allocation of seats in the Parliament and state legislatures.

RPA Act 1951 provides for

Actual conduct of elections.


Administrative machinery for conducting elections.
Poll.
Election offences.
Election disputes.
By-elections.
Registration of political parties.

Q Source: Additional Research: 10th NCERT Democratic Politics

36 Minimum Alternate Tax (MAT) may need to be paid by

A. A company even if its taxable income is shown to be ‘nil’


B. Certain foreign companies with income sources in India
C. Venture and angel funds
D. Both (a) and (b)

Correct Answer : D

Answer Justification :

Learning: Income tax is paid as per the provisions of the Income Tax Act (IT Act) but companies
calculate their profit (through profit and loss account) as per the provisions of the Companies Act.

The IT Act allows several kinds of exemptions and other incentives from total income together with
deductions on the gross income.

As a result of these exemptions, deductions and other incentives under IT Act, companies show
their taxable income either ‘nil’ or ‘negative’, and this way, the ‘zero tax’ companies emerge and
pay no taxes even though they may be paying huge dividends (under the Companies Act) to their
shareholders.

28
Total Marks : 200
Online Prelims TEST - 29 (TEXTBOOK)
( InsightsIAS Mock Test Series for UPSC Preliminary Exam 2020 ) Mark Scored : 0

To bring such companies under the income tax, MAT was imposed accordingly.

It is applicable on all companies except those engaged in infrastructure and power sectors, free
trade zones, charitable activities, venture and angel funds. Foreign companies with income sources
in India also come under it.

Q Source: Chapter on Taxation: Indian Economy: Ramesh Singh

37 Which of these nations/regions lie both in Northern and Southern Hemisphere?


1. Ecuador
2. Somalia
3. Thailand
4. Madagascar

Select the correct answer using the codes below.


A. 1 and 2 only
B. 3 and 4 only
C. 1, 2 and 3 only
D. 2, 3 and 4 only

Correct Answer : A

Answer Justification :

Learning:

29
Total Marks : 200
Online Prelims TEST - 29 (TEXTBOOK)
( InsightsIAS Mock Test Series for UPSC Preliminary Exam 2020 ) Mark Scored : 0

The image shows equator passing through countries, these are the ones that lie in both
hemispheres.

Q Source: Map-based: World map

38 The West Asian tectonic convergence includes which of these tectonic plates?
1. Indian Plate
2. African plate
3. Arabian Plate
4. Pacific plate

Select the correct answer using the codes below.


A. 2 and 3 only
B. 3 and 4 only
C. 1, 2 and 3 only
D. 2, 3 and 4 only

Correct Answer : A

Answer Justification :

Background: The outer shell of the earth, the lithosphere, is broken up into tectonic plates. The

30
Total Marks : 200
Online Prelims TEST - 29 (TEXTBOOK)
( InsightsIAS Mock Test Series for UPSC Preliminary Exam 2020 ) Mark Scored : 0

seven major plates are the African plate, Antarctic plate, Eurasian plate, Indo-Australian plate,
North American plate, Pacific plate and South American plate.

Justification: The West Asian convergence includes the African, Eurasian, and Arabian plates. The
boundaries between the tectonic plates make up the Azores-Gibraltar Ridge, extending across North
Africa, the Red Sea, and into Iran.

The Arabian Plate is moving northward into the Anatolian plate (Turkey) at the East Anatolian Fault,
and the boundary between the Aegean and Anatolian plate in eastern Turkey is also seismically
active.

Q Source: Map-based: West Asia

39 Consider the following statements.


Biotechnology Industry Research Assistance Council (BIRAC)
1. is a not-for-profit Public Sector Enterprise
2. was set up by Department of Biotechnology (DBT)
3. helps provide access to risk capital through targeted funding for the bio-tech sector

Select the correct answer using the codes below.


A. 1 only
B. 1 and 3 only
C. 2 only
D. 1, 2 and 3

Correct Answer : D

Answer Justification :

Justification: Biotechnology Industry Research Assistance Council (BIRAC) is a not-for-profit


Section 8, Schedule B, Public Sector Enterprise, set up by Department of Biotechnology (DBT),
Government of India as an Interface Agency to strengthen and empower the emerging Biotech
enterprise to undertake strategic research and innovation, addressing nationally relevant product
development needs.

BIRAC is a industry-academia interface and implements its mandate through a wide range of impact
initiatives, be it providing access to risk capital through targeted funding, technology transfer, IP
management and handholding schemes that help bring innovation excellence to the biotech firms
and make them globally competitive.

Vision

“To Stimulate, foster and enhance the strategic research and innovation capabilities of the Indian
biotech industry, particularly start-ups and SME’s, for creation of affordable products addressing
the needs of the largest section of society”

Key Strategies

31
Total Marks : 200
Online Prelims TEST - 29 (TEXTBOOK)
( InsightsIAS Mock Test Series for UPSC Preliminary Exam 2020 ) Mark Scored : 0

Foster innovation and entrepreneurship


Promote affordable innovation in key social sectors
Empowerment of start-ups & small and medium enterprises
Contribute through partners for capability enhancement and diffusion of innovation
Enable commercialization of discovery
Ensure global competitiveness of Indian enterprises

Q Source: http://www.birac.nic.in/desc_new.php?id=89

40 India Justice Report (IJR) was released by

A. Association for Democratic Reforms (ADR)


B. National Human Rights Commission (NHRC)
C. Commonwealth Forum of National Human Rights
D. Tata Trusts

Correct Answer : D

Answer Justification :

Justification: India Justice Report (IJR) was prepared by Tata Trusts in collaboration with Centre
for social Justice, Common Cause, Commonwealth Human Rights Initiative, Daksh, TISS-Prayas and
Vidhi Centre for Legal Policy. It is India's first-ever ranking of states on justice delivery.

Performance of various states: The list is topped by Maharashtra. Maharashtra is followed by Kerala
and Tamil Nadu.

Key Issues highlighted:

1. Vacancy was an issue across the pillars of the police, prisons, and the judiciary, with only about
half the states having made the effort to reduce these over a five-year period.

2. The country as a whole has about 18,200 judges with about 23 per cent sanctioned posts vacant.

3. By 2017, women are also poorly represented in these pillars.

4. The prisons in the country were over occupied at 114 per cent, where 68 per cent are undertrials
awaiting investigation, inquiry or trial.

5. Nationally, high vacancies in the justice system.

6. Only two states have met the 80 per cent of SC/ST/OBC reservation required to be followed and
that there are high vacancies in prison staff.

Q Source:
https://www.insightsonindia.com/wp-content/uploads/2019/12/Insights-November-2019-Current-Affa

32
Total Marks : 200
Online Prelims TEST - 29 (TEXTBOOK)
( InsightsIAS Mock Test Series for UPSC Preliminary Exam 2020 ) Mark Scored : 0

irs-Compilation.pdf

41 Consider the following statements about Currency Swap Arrangement (CSA).


1. It is a multilateral agreement where member countries adhere to a ‘single currency’ for all trading
and transactions purposes with all other member countries.
2. Exchange rates are usually predetermined by countries in a CSA.
3. India has no framework on currency swap arrangement with SAARC countries till date.

Select the correct answer using the codes below.


A. 1, 2 and 3
B. 1 and 3 only
C. 2 only
D. 2 and 3 only

Correct Answer : C

Answer Justification :

Justification: Statement 1 and 2: A Currency Swap Arrangement (CSA) is an arrangement,


between two friendly countries, which have regular, substantial or increasing trade, to basically
involve in trading in their own local currencies, where both pay for import and export trade, at the
predetermined rates of exchange, without bringing in third country currency like the US Dollar. In
such arrangements no third country currency is involved, thereby eliminating the need to worry
about exchange variations.

Statement 3: With an objective to strengthen financial stability and economic cooperation, the
Reserve Bank of India has revised the framework on currency swap arrangement for SAARC
countries till 2022. As per the new framework:

1. RBI will continue to offer swap arrangement within the overall corpus of $2 billion.

2. RBI would enter into bilateral swap agreements with SAARC central banks, who want to avail
swap facility.

3. The drawals can be made in US dollar, euro or Indian rupee.

4. The currency swap facility will be available to all SAARC member countries, subject to their
signing the bilateral swap agreements.

Learning: Significance of the agreement:

1. The currency swap agreement is an important measure in improving the confidence in the Indian
market and it would not only enable the agreed amount of capital being available to India, but it will
also bring down the cost of capital for Indian entities while accessing the foreign capital market.

2. The swap arrangement should aid in bringing greater stability to foreign exchange and capital

33
Total Marks : 200
Online Prelims TEST - 29 (TEXTBOOK)
( InsightsIAS Mock Test Series for UPSC Preliminary Exam 2020 ) Mark Scored : 0

markets in India. With this arrangement in place, prospects of India would further improve in
tapping foreign capital for country’s developmental needs. This facility will enable the agreed
amount of foreign capital being available to India for use as and when the need arises.

Q Source: Frequently in news

42 Census of India is conducted by the Office of the Registrar General and Census Commissioner, which
is under

A. Ministry of Home Affairs


B. Ministry of Statistics and Programme Implementation
C. Ministry of Planning
D. Ministry of Social Justice and Empowerment

Correct Answer : A

Answer Justification :

Learning: The responsibility of conducting the decennial Census rests with the Office of the
Registrar General and Census Commissioner, India under Ministry of Home Affairs.

The Census Organisation was set up on an ad-hoc basis for each Census till the 1951 Census.

The Census Act was enacted in 1948 to provide for the scheme of conducting population census
with duties and responsibilities of census officers.

Q Source: Census of India

43 K12 Education Transformation Framework, launched by Microsoft, facilitates


1. Digital transformation of schools
2. Connecting institutions of higher learning with a high-speed optical fibre network

Select the correct answer using the codes below.


A. 1 only
B. 2 only
C. Both 1 and 2
D. None of the above

Correct Answer : A

Answer Justification :

Justification: Microsoft rolled out its K12 Education Transformation Framework more widely in

34
Total Marks : 200
Online Prelims TEST - 29 (TEXTBOOK)
( InsightsIAS Mock Test Series for UPSC Preliminary Exam 2020 ) Mark Scored : 0

India.

Key features of the framework:

1. The Framework provides a holistic framework to facilitate a comprehensive digital


transformation of schools.

2. The framework is a flexible platform based on the latest research and input from hundreds of
academics, experts, and policymakers.

3. It comprises four pillars — leadership and policy, modern teaching and learning, intelligent
environments and technology blueprint.

Education leaders in more than 50 countries have already adopted the Microsoft K-12 Education
Transformation Framework to help plan their learning strategies in partnership with Microsoft.

Q Source: November 2019 Current Affairs: Insights

44 Consider the following statements.


The Treaty of Allahabad granted rights to the British to
1. Collect taxes from the eastern province of Bengal-Bihar-Orissa
2. Keep an armed contingent on behalf of the Bengal nawab

Which of the above is/are correct?


A. 1 only
B. 2 only
C. Both 1 and 2
D. None

Correct Answer : A

Answer Justification :

Justification: It was signed in 1765, between the Mughal Emperor Shah Alam II and Robert, Lord
Clive, as a result of the Battle of Buxar.

The Treaty marks the political and constitutional involvement and the beginning of British
rule in India.
Based on the terms of the agreement, Alam granted the East India Company Diwani rights, or
the right to collect taxes on behalf of the Emperor from the eastern province of Bengal-Bihar-
Orissa.
Thus, East India Company got appointed as the imperial tax collector for the Eastern province
(Bengal-Bihar-Orissa).
These rights allowed the Company to collect revenue directly from the people of Bengal, Bihar
and Orissa.

Q Source: Major battles that border Medieval and Modern India

35
Total Marks : 200
Online Prelims TEST - 29 (TEXTBOOK)
( InsightsIAS Mock Test Series for UPSC Preliminary Exam 2020 ) Mark Scored : 0

45 Consider the following about the Treaty on the Prohibition of Nuclear Weapons.
1. It bans and makes it illegal to possess or deploy nuclear weapons.
2. None of the nuclear weapons nations have signed the treaty.
3. It will not come in force if not ratified by at least two-thirds of the members of the UN.

Select the correct answer using the codes below.


A. 1 and 2 only
B. 2 and 3 only
C. 1 only
D. 1, 2 and 3

Correct Answer : A

Answer Justification :

Justification: Statement 1: It makes it illegal to possess, use, produce, transfer, acquire, stockpile
or deploy nuclear weapons.

This was the culmination of 10 years of preparation by many national and international
organizations.

Statement 2: It was signed and approved by 122 of the 123 participant nations, representing two-
thirds of the nations in the UN. Interestingly, none of the nuclear weapons nations participated. As
of 23 January 2020, 35 states have ratified the treaty.

Statement 3: This is the most significant multilateral development on nuclear arms control since
the adoption of the Nuclear Non-Proliferation Treaty (NPT) in 1968. It has to be ratified by 50
countries to come into force.

Q Source: https://www.un.org/disarmament/wmd/nuclear/tpnw/

46 Consider the following about the ASEAN Defence Ministers’ Meeting Plus (ADMM-Plus).
1. It is an informal group consisting of ASEAN nations only.
2. The group agrees to cooperate on a variety of issues, inter alia, maritime security, counterterrorism,
humanitarian assistance and disaster relief.

Select the correct answer using the codes below.


A. 1 only
B. 2 only
C. Both 1 and 2
D. None of the above

Correct Answer : B

36
Total Marks : 200
Online Prelims TEST - 29 (TEXTBOOK)
( InsightsIAS Mock Test Series for UPSC Preliminary Exam 2020 ) Mark Scored : 0

Answer Justification :

Justification: The ADMM-Plus is a platform for ASEAN and its eight Dialogue Partners, namely
Australia, China, India, Japan, New Zealand, ROK, Russian Federation and the United States, to
strengthen security and defence cooperation for peace, stability, and development in the region.
The Inaugural ADMM-Plus was convened in Ha Noi, Viet Nam, on 12 October 2010.

The objectives of the ADMM-Plus, are as follows:

To benefit ASEAN member countries in building capacity to address shared security


challenges, while cognisant of the differing capacities of various ASEAN countries;
To promote mutual trust and confidence between defence establishments through greater
dialogue and transparency;
To enhance regional peace and stability through cooperation in defence and security, in view
of the transnational security challenges the region faces;
To contribute to the realisation of an ASEAN Security Community which, as stipulated in the
Bali Concord II, embodies ASEAN’s aspiration to achieve peace, stability, democracy and
prosperity in the region where ASEAN member countries live at peace with one another and
with the world at large;
To facilitate the implementation of the Vientiane Action Programme, which calls for ASEAN to
build a peaceful,secure and prosperous ASEAN, and to adopt greater outward-looking
external relation strategies with our friends and Dialogue Partners.

The ADMM-Plus currently focuses on seven areas of practical cooperation, namely maritime
security, counter-terrorism, humanitarian assistance and disaster management, peacekeeping
operations, military medicine, humanitarian mine action and cyber security. Experts Working
Groups (EWGs) have been established to facilitate cooperation in these areas.

Q Source:
https://asean.org/asean-political-security-community/asean-defence-ministers-meeting-admm/

47 UN convention on the Rights of persons with disabilities includes which of these rights for the
differently abled?
1. Rights to accessibility including the information technology
2. Rights to live independently and be included in the community
3. Rights to habitation and rehabilitation

Select the correct answer using the codes below.


A. 1 only
B. 2 and 3 only
C. 2 only
D. 1, 2 and 3

Correct Answer : D

37
Total Marks : 200
Online Prelims TEST - 29 (TEXTBOOK)
( InsightsIAS Mock Test Series for UPSC Preliminary Exam 2020 ) Mark Scored : 0

Answer Justification :

Justification: Rights specific to this convention include the rights to accessibility including the
information technology, the rights to live independently and be included in the community (Article
19), to personal mobility (article 20), habilitation and rehabilitation (Article 26), and to participation
in political and public life, and cultural life, recreation and sport (Articles 29 and 30).

Learning: Articles 33–39 govern reporting and monitoring of the convention by national human
rights institutions (Article 33) and Committee on the Rights of Persons with Disabilities (Article 34).

Q Source:
https://www.un.org/development/desa/disabilities/convention-on-the-rights-of-persons-with-disabiliti
es.html

48 Consider the following statements.


1. The term Gram Sabha is defined in the Constitution of India.
2. Gram Sabha is a permanent body and not subject to dissolution.
3. The power to annul a decision of the Gram Sabha rests with the Gram Sabha only.

Select the correct answer using the codes below.


A. 1 and 3 only
B. 2 only
C. 1, 2 and 3
D. 2 and 3 only

Correct Answer : C

Answer Justification :

Justification: The term Gram Sabha is defined in the Constitution of India under Article 243(b).
Gram Sabha is the primary body of the Panchayati Raj system and by far the largest. It is a
permanent body. The power to annul a decision of the Gram Sabha rests with the Gram Sabha only.

Composition:

1. Persons, those who are above 18 years of age.

2. Living in the village.

3. Whose names are included in the electoral rolls for the Panchayat at the village level.

Powers and functions: Constitution mentions that Gram Sabha exercises such powers and performs
such functions at the village level as the Legislature of a State may, by law, provide

Learning: Important and specific functions of Gram Sabha:

1. To help implementation of the development programmes and schemes of the Panchayat.

38
Total Marks : 200
Online Prelims TEST - 29 (TEXTBOOK)
( InsightsIAS Mock Test Series for UPSC Preliminary Exam 2020 ) Mark Scored : 0

2. To identify beneficiaries for different programmes and schemes. However, if the Gram Sabha fails
to identify such beneficiaries within a reasonable time, the Gram Panchayat shall identify the
beneficiaries.

3. To solicit support — in cash or kind or both and voluntary labour — from the public for
community welfare programmes.

4. To support the programmes of mass education and family welfare.

5. To promote unity and harmony among all sections of the society in the village.

6. To seek clarification from the Mukhiya, Up-Mukhiya and other members of the Gram Panchayat
about any particular activity, scheme, income and expenditure.

7. To discuss and recommend appropriate action with regard to reports of the Vigilance Committee.

8. Other related matters brought to the notice of the Gram Sabha.

9. To consider levy of taxes, rates, rents & fees & enhancement of rates thereof.

10. To consider all such matters as may be referred by the Gram Panchayat for its decision.

Q Source: Basics: Polity

49 The aim of Integrated Automatic Aviation Meteorological System (IAAMS), recently seen in news, is
to
1. Bring accuracy in weather monitoring mechanisms in Indian Navy
2. Replace the old aeronautical navigation systems in the Indian Air force
3. Provide troop movement support to the Indian Armed Forces in extreme weather conditions on
tough terrains

Select the correct answer using the codes below.


A. 1 only
B. 2 and 3 only
C. 1 and 3 only
D. 1 and 2 only

Correct Answer : A

Answer Justification :

Justification: IAAMS is expected to modernise the meteorological infrastructure of the nine Naval
Air Stations (NAS).

IAAMS is equipped with the state-of-the-art meteorological sensors.


It undertakes automatic and continuous recording of relevant weather parameters that are
vital for accurate weather forecasting.
It has a special alarm feature that alerts the duty staff during abnormal change of weather

39
Total Marks : 200
Online Prelims TEST - 29 (TEXTBOOK)
( InsightsIAS Mock Test Series for UPSC Preliminary Exam 2020 ) Mark Scored : 0

parameters that may affect safe flying operations.


It can also provide automatic dissemination of routine weather reports as per standards of
World Meteorological Organisation (WMO) to other air stations and Air Traffic Controller
towers without human intervention.
The IAAMS project at INS Garuda will give a major fillip to aviation safety through automation
of weather monitoring process.

Q Source:
https://www.indiannavy.nic.in/content/inauguration-integrated-automatic-aviation-meteorological-sy
stem-iaams-ins-parundu

50 Consider the following statements about the economic developments post-2014 as presented in
Economic Survey 2019-20.
1. India ranks first in the number of new firms created.
2. New firm creation in services sector is higher than that in manufacturing, infrastructure or
agriculture sector.

Select the correct answer using the codes below.


A. 1 only
B. 2 only
C. Both 1 and 2
D. None of the above

Correct Answer : B

Answer Justification :

Justification: Chapter 2 of the Survey examines the content and drivers of entrepreneurial activity
at the bottom of the administrative pyramid – over 500 districts in India.

The analysis employs comprehensive data on new firm creation in the formal sector across all these
districts from the Ministry of Corporate Affairs (MCA)-21 database.

First, using the World Bank’s Data on Entrepreneurship, this chapter confirms that India ranks
third in number of new firms created. The same data shows that new firm creation has gone up
dramatically in India since 2014.

While the number of new firms in the formal sector grew at a compounded annual growth rate
of 3.8 per cent from 2006-2014, the growth rate from 2014 to 2018 has been 12.2 per cent. As
a result, from about 70,000 new firms created in 2014, the number has grown by about 80 per
cent to about 1,24,000 new firms in 2018.

Second, reflecting India’s new economic structure, i.e. comparative advantage in the Services
sector, new firm creation in services is significantly higher than that in manufacturing,
infrastructure or agriculture.

Third, grassroots entrepreneurship is not just driven by necessity as a 10 percent increase in

40
Total Marks : 200
Online Prelims TEST - 29 (TEXTBOOK)
( InsightsIAS Mock Test Series for UPSC Preliminary Exam 2020 ) Mark Scored : 0

registration of new firms in a district yields a 1.8 percent increase in GDDP.

Fourth, birth of new firms is very heterogeneous across Indian districts and across sectors.
Moreover, it is dispersed across India and is not restricted to just a few cities.

Q Source: Ch 2: Economic Survey 2019-20

51 In terms of volume of production, among the following, the most produced crop during 2019-20 was

A. Sugarcane
B. Oil seeds
C. Pulses
D. Cotton

Correct Answer : A

Answer Justification :

Learning: Total production of Sugarcane in the country during 2019-20 is estimated at


353.85 million tonnes. The production of sugarcane during 2019-20 is higher by 4.07 million
tonnes than the average sugarcane production of 349.78 million tonnes.

Total Oilseeds production in the country during 2019-20 is estimated at 34.19 million tonnes which
is higher by 2.67 million tonnes than the production of 31.52 million tonnes during 2018-19.
Further, the production of oilseeds during 2019-20 is higher by 4.54 million tonnes than the average
oilseeds production.

Production of Cotton is estimated at 34.89 million bales (of 170 kg each) is higher by 6.85 million
bales than the production of 28.04 million bales during 2018-19. Production of Jute & Mesta is
estimated at 9.81 million bales (of 180 kg each).

Total production of rice during 2019-20 is estimated at record 117.47 million tonnes. It is higher by
9.67 million tonnes than the five years’ average production of 107.80 million tonnes.

Production of wheat during 2019-20 is estimated at record 106.21 million tonnes. It is higher by
2.61 million tonnes as compared to wheat production during 2018-19 and is higher by 11.60 million
tonnes than the average wheat production of 94.61 million tonnes.

Production of Nutri / Coarse Cereals estimated at 45.24 million tonnes, which is higher by 2.18
million tonnes than the production of 43.06 million tonnes achieved during 2018-19. Further, it is
also higher by 2.16 million tonnes than the average production.

Total Pulses production during 2019-20 is estimated at 23.02 million tonnes which is higher by 2.76
million tonnes than the Five years’ average production of 20.26 million tonnes.

Q Source:
https://economictimes.indiatimes.com/news/economy/agriculture/indias-2019-20-foodgrain-producti
on-to-hit-a-record-high-of-291-95-million-tonnes-estimates-second-advance-estimate-of-

41
Total Marks : 200
Online Prelims TEST - 29 (TEXTBOOK)
( InsightsIAS Mock Test Series for UPSC Preliminary Exam 2020 ) Mark Scored : 0

govt/articleshow/74192668.cms?from=mdr

52 ASEAN+3 group includes which of these nations apart from ASEAN nations?
1. India
2. USA
3. China
4. Japan
5. South Korea

Select the correct answer using the codes below.


A. 1, 3 and 4 only
B. 2, 4 and 5 only
C. 3, 4 and 5 only
D. 1, 2 and 5 only

Correct Answer : C

Answer Justification :

Justification: ASEAN+3 group includes ASEAN plus China, Japan, and South Korea.

ASEAN + 6 is an economic partnership of the members of the Association of Southeast Asian


Nations plus Australia, China, India, Japan, New Zealand, and South Korea.

It is intended to accelerate economic growth in East Asian countries, promote cooperation in


energy, foods, and other fields.

Q Source: Frequently in news

53 Consider the following definitions.


1. A “carbon offset” is an electronic and serialized unit that represents one kg of CO2 equivalent that
is reduced, avoided, or sequestered from projects applying an approved carbon credit methodology.
2. The IPCC defines carbon neutrality as “the complete elimination of carbon emissions (carbon
dioxide).”
3. Decarbonization refers to the conversion of the economic system or individual carbon emitting
entity reducing the carbon intensity of its (direct or value chain) emissions over time.

Select the correct answer using the codes below.


A. 1 and 2 only
B. 2 and 3 only
C. 3 only
D. 1, 2 and 3

42
Total Marks : 200
Online Prelims TEST - 29 (TEXTBOOK)
( InsightsIAS Mock Test Series for UPSC Preliminary Exam 2020 ) Mark Scored : 0

Correct Answer : C

Answer Justification :

Justification: Statement 1: A “carbon credit” (also known as a “carbon offset”) is an electronic and
serialized unit that represents one ton of CO2 equivalent that is reduced, avoided, or sequestered
from projects applying an approved carbon credit methodology.

Related terms:

CO2 equivalent is a unit of measurement used to express the global warming potential of each
different greenhouse gas in terms of the amount of CO2 that would create the same amount of
warming.
Carbon crediting program: A program under which emissions reduction projects are certified
and issued carbon credits. Examples include programs typically used for emissions
compliance obligations, such as the Clean Development Mechanism (CDM), and programs
typically or exclusively used for voluntary carbon credit purchases, such as the Gold Standard,
Verified Carbon Standard, Climate Action Reserve, American Carbon Registry, and Plan Vivo.

Statement 2: The IPCC defines carbon neutrality as “balancing of residual emissions with emission
(carbon dioxide) removal.” This is not how the term is used in the marketplace, however. Many will
define carbon neutrality as when an entity is balancing out carbon emissions it has caused by
funding an equivalent amount of carbon savings elsewhere in the world. Carbon neutrality can
sometimes be misperceived as meaning only the complete elimination of emissions.

Statement 3: Literally, it means the reduction of carbon. More specifically, the term refers to the
conversion of the economic system or individual carbon emitting entity converting to reduce the
carbon intensity of its (direct or value chain) emissions over time.

Q Source: Read more important terminologies here:


https://c402277.ssl.cf1.rackcdn.com/publications/1310/files/original/WWF_position_and_guidance_o
n_corporate_use_of_voluntary_carbon_credits_EXTERNAL_VERSION_11_October_2019.pdf?158384
6796

54 In Ancient India, officers who reported the activities in the country to the government were known as

A. Agraharas
B. Dhamma Mahathas
C. Patidevakas
D. Gramani

Correct Answer : C

Answer Justification :

Learning: The duty of these officers was to superintend or oversee all that occurred in town or

43
Total Marks : 200
Online Prelims TEST - 29 (TEXTBOOK)
( InsightsIAS Mock Test Series for UPSC Preliminary Exam 2020 ) Mark Scored : 0

country and to make private reports to the government.

Arian notes that similar officers were employed by the authorities of the independent nations as
well as by the monarchical governments of India.

King Devanampiya says, “the patidevakas should report to me about the affairs of the people at all
times, anywhere, whether I am eating, in the inner apartment, in the bedroom or in the garden. And
I will dispose of the affairs of the people everywhere.”

Q Source: Revision: Chapter 2: 12th NCERT: Themes in Indian History – I

55 Consider the following about Science Based Targets initiative (SBTi).


1. It is a collaboration between United Nations Environment Program (UNEP) and UNICEF.
2. It aims to institutionalize corporate emissions reduction target setting so that their reduction
targets are scientifically in line with the global targets to limit global warming.

Select the correct answer using the codes below.


A. 1 only
B. 2 only
C. Both 1 and 2
D. None of the above

Correct Answer : B

Answer Justification :

Justification: Targets adopted by companies to reduce greenhouse gas (GHG) emissions are
considered “science-based” if they are in line with what the latest climate science says is necessary
to meet the goals of the Paris Agreement—to limit global warming to well-below 2°C above pre-
industrial levels and pursue efforts to limit warming to 1.5°C.

The Science Based Targets initiative champions science-based target setting as a powerful way of
boosting companies’ competitive advantage in the transition to the low-carbon economy. It is a
collaboration between CDP, World Resources Institute (WRI), the World Wide Fund for Nature
(WWF), and the United Nations Global Compact (UNGC). It is one of the We Mean Business
Coalition commitments.

The initiative:

1. Showcases companies that set science-based targets through case studies, events and media
to highlight the increased innovation, reduced regulatory uncertainty, strengthened investor
confidence and improved profitability and competitiveness generated by science-based target
setting.
2. Defines and promotes best practice in science-based target setting with the support of a
Technical Advisory Group
3. Offers resources, workshops and guidance to reduce barriers to adoption
4. Independently assesses and approves companies’ targets .

44
Total Marks : 200
Online Prelims TEST - 29 (TEXTBOOK)
( InsightsIAS Mock Test Series for UPSC Preliminary Exam 2020 ) Mark Scored : 0

Q Source: https://sciencebasedtargets.org/faq/

https://c402277.ssl.cf1.rackcdn.com/publications/1310/files/original/WWF_position_and_guidance_o
n_corporate_use_of_voluntary_carbon_credits_EXTERNAL_VERSION_11_October_2019.pdf?158384
6796

56 Which of these protected areas is both a World heritage and a Ramsar site?

A. Betla National Park


B. Dachigam National Park
C. Keoladeo National park
D. Sanjay Gandhi National Park

Correct Answer : C

Answer Justification :

Learning: The protected area, located in Western India, is one of the richest bird areas in the world
and is known for nesting of resident birds and visiting migratory birds including water birds. It is
also a Ramsar site and a World heritage site.

Every year thousands of migratory waterfowl visit the park for wintering and breeding.

The reserve protects Bharatpur from frequent floods, provides grazing grounds for village cattle,
and earlier was primarily used as a waterfowl hunting ground.

We have covered the park in a previous test.

Q Source: Important protected areas

57 With reference to Horse latitudes, consider the following.


1. It is situated between the tropics.
2. It is common in this region for winds to diverge and always flow towards the equator creating a
region of perennial low pressure.

Select the correct answer using the codes below.


A. 1 only
B. 2 only
C. Both 1 and 2
D. None

Correct Answer : D

Answer Justification :

45
Total Marks : 200
Online Prelims TEST - 29 (TEXTBOOK)
( InsightsIAS Mock Test Series for UPSC Preliminary Exam 2020 ) Mark Scored : 0

Justification: The horse latitudes are located at about 30 degrees north and south of the equator.

It is common in this region of the subtropics for winds to diverge and either flow toward the
poles (known as the prevailing westerlies) or toward the equator (known as the trade winds).
These diverging winds are the result of an area of high pressure, which is characterized by
calm winds, sunny skies, and little or no precipitation.
According to legend, the term comes from ships sailing to the New World that would often
become stalled for days or even weeks when they encountered areas of high pressure and
calm winds.
Many of these ships carried horses to the Americas as part of their cargo. Unable to sail and
resupply due to lack of wind, crews often ran out of drinking water.
To conserve scarce water, sailors on these ships would sometimes throw the horses they were
transporting overboard. Thus, the phrase 'horse latitudes' was born.

Q Source: https://oceanservice.noaa.gov/facts/horse-latitudes.html

58 Consider the following statements.


1. Freshwater ecosystems cover less than 0.01% of the planet’s total surface area.
2. More than half of the world’s wetlands have disappeared since the beginning of the 20th CE.

Select the correct answer using the codes below.


A. 1 only
B. 2 only
C. Both 1 and 2
D. None of the above

Correct Answer : C

Answer Justification :

Justification: Freshwater ecosystems cover less than 0.01% of the planet’s total surface area but
they support more than 125,000 species, from fish to terrestrial animals.

46
Total Marks : 200
Online Prelims TEST - 29 (TEXTBOOK)
( InsightsIAS Mock Test Series for UPSC Preliminary Exam 2020 ) Mark Scored : 0

They also provide a myriad of ecosystem services, including helping to regulate the temperature of
the land and sea, creating clouds and affecting the weather, transporting nutrients and minerals,
and keeping nature clean and healthy by dissolving pollutants and sediment run off.

However, more than half of the world’s wetlands have disappeared since 1900, and fewer than 70 of
the world’s 177 longest rivers remain free of man-made obstructions.

Q Source: https://www.worldwildlife.org/initiatives/fresh-water

59 The Heavy Mineral Sand deposits in Kerala contain an assemblage of which of these minerals
1. Ilmenite
2. Zircon
3. Monazite

Select the correct answer using the codes below.


A. 1 and 2 only
B. 2 only
C. 3 only
D. 1, 2 and 3

Correct Answer : D

Answer Justification :

Justification: Kerala State is endowed with a number of occurrences/deposits of minerals such as


Heavy Mineral Sands ( Ilmenite, Rutile, Zircon, Monazite, Sillimanite) ,Gold, Iron ore, Bauxite,
Graphite, China Clay, Fire Clay, Tile and Brick Clay, Silica Sand, Lignite, Limestone, Limeshell,
Dimension Stone (Granite), Gemstones, Magnesite, Steatite etc.

However, mining activities on large scale are confined mainly to a few minerals - Heavy Mineral
Sands, China Clay and to a lesser extent Limestone/Limeshell, Silica Sand and Granite.

In fact, Heavy mineral sand and China Clay contribute more than 90% of the total value of mineral
production in the State.

Q Source: Based on CSP papers

60 Sea turtles, globally, can be found in


1. Shallow seagrass beds in the Indian Ocean
2. Reefs in ‘Coral Triangle’ (Western Pacific)
3. Beaches of Eastern Pacific Ocean

Select the correct answer using the codes below.


A. 1 only
B. 2 and 3 only
C. 1, 2 and 3

47
Total Marks : 200
Online Prelims TEST - 29 (TEXTBOOK)
( InsightsIAS Mock Test Series for UPSC Preliminary Exam 2020 ) Mark Scored : 0

D. 2 only

Correct Answer : C

Answer Justification :

Justification: Seven different species of sea (or marine) turtles grace our ocean waters, from the
shallow seagrass beds of the Indian Ocean, to the colorful reefs of the Coral Triangle and the sandy
beaches of the Eastern Pacific. While these highly migratory species periodically come ashore to
either bask or nest, sea turtles spend the bulk of their lives in the ocean. WWF's work on sea turtles
focuses on five of those species: green, hawksbill, loggerhead, leatherback, and olive ridley.

The Coral Triangle is a roughly triangular area of the tropical marine waters of Indonesia, Malaysia,
Papua New Guinea, Philippines, Solomon Islands and Timor-Leste that contain at least 500 species
of reef-building corals in each ecoregion.

Over the last 200 years, human activities have tipped the scales against the survival of these ancient
mariners. Slaughtered for their eggs, meat, skin, and shells, sea turtles suffer from poaching and
over-exploitation.

They also face habitat destruction and accidental capture—known as bycatch—in fishing gear.
Climate change has an impact on turtle nesting sites; it alters sand temperatures, which then
affects the sex of hatchlings. Nearly all species of sea turtle are now classified as endangered, with
three of the seven existing species being critically endangered.

Q Source: https://www.worldwildlife.org/species/sea-turtle

61 Consider the following statements.


1. India is the largest exporter of commercial services in the World.
2. India’s total foreign trade accounts for about one-fifths of its GDP.

Select the correct answer using the codes below.


A. 1 only
B. 2 only
C. Both 1 and 2
D. None

Correct Answer : D

Answer Justification :

Justification: Statement 1: Trade accounts for nearly 35-40% of our GDP. Total Trade is about 950
billion USD and India’s GDP is 2597 billion USD.

Foreign trade accounted for 48.8% of India's GDP in 2017.

48
Total Marks : 200
Online Prelims TEST - 29 (TEXTBOOK)
( InsightsIAS Mock Test Series for UPSC Preliminary Exam 2020 ) Mark Scored : 0

Statement 2: India was the eighth largest exporter of commercial services in the world in 2016,
accounting for 3.4% of global trade in services. India recorded a 5.7% growth in services trade in
2016–17.

Q Source: Trade statistics

62 Consider the following statements.


1. National Ganga Council is under the chairmanship of Union Minister Union Minister of Jal Shakti.
2. National Mission for Clean Ganga (NMCG) currently is an implementation arm of National Ganga
River Basin Authority (NGRBA).

Select the correct answer using the codes below.


A. 1 only
B. 2 only
C. Both 1 and 2
D. None of the above

Correct Answer : D

49
Total Marks : 200
Online Prelims TEST - 29 (TEXTBOOK)
( InsightsIAS Mock Test Series for UPSC Preliminary Exam 2020 ) Mark Scored : 0

Answer Justification :

Justification: National Mission for Clean Ganga (NMCG) was registered as a society on 12th
August 2011 under the Societies Registration Act 1860. It acted as implementation arm of National
Ganga River Basin Authority (NGRBA) which was constituted under the provisions of the
Environment (Protection) Act (EPA),1986. NGRBA has since been dissolved with effect from the 7th
October 2016, consequent to constitution of National Council for Rejuvenation, Protection and
Management of River Ganga (referred as National Ganga Council).

The Act envisages five tier structure at national, state and district level to take measures for
prevention, control and abatement of environmental pollution in river Ganga and to ensure
continuous adequate flow of water so as to rejuvenate the river Ganga as below;

1. National Ganga Council under chairmanship of Hon’ble Prime Minister of India.

2. Empowered Task Force (ETF) on river Ganga under chairmanship of Hon’ble Union Minister
of Jal Shakti (Department of Water Resources, River Development and Ganga Rejuvenation).

3. National Mission for Clean Ganga(NMCG).

4. State Ganga Committees and

5. District Ganga Committees in every specified district abutting river Ganga and its tributaries
in the states.

Q Source: https://nmcg.nic.in/about_nmcg.aspx

63 Consider the following statements.


1. India presently follows the International Financial Reporting Standards (IFRS) developed by the
Financial Stability Board (FSB).
2. Companies and Banks in India have been directed to follow the Indian Accounting Standards by the
Ministry of Finance.

Which of the above is/are correct?


A. 1 only
B. 2 only
C. Both 1 and 2
D. None

Correct Answer : D

Answer Justification :

Justification: Statement 1: Banks and non-banking financial companies currently follow Indian
generally accepted accounting principles (GAAP) standards. Other corporate entities started
complying with IndAS with effect from 1 April 2016.

The Financial Accounting Standards Board (FASB) issues GAAP. The international alternative to

50
Total Marks : 200
Online Prelims TEST - 29 (TEXTBOOK)
( InsightsIAS Mock Test Series for UPSC Preliminary Exam 2020 ) Mark Scored : 0

GAAP is the International Financial Reporting Standards (IFRS)

Statement 2: Since Companies already follow IndAS, it is the banks, especially Public Sector Banks
that need to follow IndAS. Ind AS or Indian Accounting Standards govern the accounting and
recording of financial transactions as well as the presentation of statements such as profit and loss
account and balance sheet of a company.

Learning: Indian Accounting Standard (abbreviated as Ind-AS) is the Accounting standard adopted
by companies in India and issued under the supervision and control of Accounting Standards Board
(ASB).

For long, there has been a heated debate about Indian companies moving to the globally
accepted International Financial Reporting Standards (IFRS) for their accounts.
But firms have resisted this shift, stating that this will lead too many changes in the capture
and reporting of their numbers. Ind AS has been evolved as a compromise formula that tries
to harmonise Indian accounting rules with the IFRS.
International Financial Reporting Standards (IFRS) is a set of accounting standards,
developed by the International Accounting Standards Board (IASB), that is becoming the
global standard for the preparation of public company financial statements. The IASB is an
independent accounting standards body, based in London

Q Source: Based on past year UPSC papers

64 Which of the following tribes can be found in Southern India?


1. Malaimalasars
2. Eravalar
3. Muduvars
4. Pulaiyars

Select the correct answer using the codes below.


A. 1 and 3 only
B. 2 and 4 only
C. 1, 2 and 3 only
D. 1, 2, 3 and 4

Correct Answer : D

Answer Justification :

Justification: The Annamalai Reserve has significant anthropological diversity with more than
4600 Adivasi people from six tribes of indigenous people living in 34 settlements.

The tribes are the Kadars, Malasars, Malaimalasars, Pulaiyars, Muduvars and the Eravallan
(Eravalar)

Kadar reside along the hilly border between Cochin in the state of Kerala and Coimbatore in the
state of Tamil Nādu.

51
Total Marks : 200
Online Prelims TEST - 29 (TEXTBOOK)
( InsightsIAS Mock Test Series for UPSC Preliminary Exam 2020 ) Mark Scored : 0

The Kadar live in the forests and do not practice agriculture, building shelters thatched with
leaves and shifting location as their employment requires. They prefer to eat rice obtained in
trade or as wages rather than to subsist on food of their own gathering.
They have long served as specialized collectors of honey, wax, sago, cardamom, ginger, and
umbrella sticks for trade with merchants from the plains. Many Kadar men work as labourers.

The Malasar speak to each other in a mixture of the Malayam and Tamil languages. They eat wild
yams and rice.

They cultivate ragi and many other kinds of millets. The Malasar live in hamlets called Pathis and
prefer to live in the plains

Q Source: Based on CSP papers

65 Apart from Teesta, India and Bangladesh have been engaged in discussions to potentially share the
water of which of these rivers?
1. Gumti
2. Manu
3. Feni
4. Khowai

Select the correct answer using the codes below.


A. 2 and 3 only
B. 1 and 4 only
C. 1, 2 and 3 only
D. 1, 2, 3 and 4

Correct Answer : D

Answer Justification :

Learning: Discussions have been continuing with Bangladesh for sharing of waters of Teesta &
Feni rivers besides other six common rivers namely; Manu, Muhri, Khowai, Gumti, Jaldhaka and
Torsa. Govt. of India is at its endeavour to conclude the agreement of the sharing of waters of
Teesta and Feni rivers with Bangladesh, which is acceptable to all parties concerned and which
protects the interests of all stakeholders.

There exists a system of Transmission of flood forecasting data on major rivers like Ganga, Teesta,
Brahmputra and Barak during the monsoon season from India to Bangladesh. The transmission of
flood forecasting information during the monsoon has enabled the civil and military authorities in
Bangladesh to shift the population affected by floods to safer places.

Also, an Indo-Bangladesh Joint Rivers Commission (JRC) is functioning since 1972. It was
established with a view to maintain liaison in order to ensure the most effective joint effort in
maximizing the benefits from common river systems. The JRC is headed by Water Resources

52
Total Marks : 200
Online Prelims TEST - 29 (TEXTBOOK)
( InsightsIAS Mock Test Series for UPSC Preliminary Exam 2020 ) Mark Scored : 0

Ministers of both the countries.

Q Source:
http://mowr.gov.in/international-cooperation/bilateral-cooperation-with-neighbouring-countries/indo
-bangladesh-cooperation

66 His rule is significant for the introduction of token currency. He took keen interest to circulate gold
and silver coins and introduced the gold coin as Dinar in India. He was?

A. Tughlaq
B. Mir Kasim
C. Aurangjeb
D. Khiliji

Correct Answer : A

Answer Justification :

Learning: He understood the importance of currency as a medium of commercial exchange and


that is why he took keen interest to circulate gold and silver coins.

The gold coin was introduced as Dinar. Tughlaq’s silver coin was named Adl. However, it was
difficult to maintain the supply of gold and silver coins on a large scale.

So, Tughlaq replaced those coins and started the circulation of copper and brass coins as the token
currency which had the same value of gold or silver coins in 1330-32 CE.

He was well aware that the state had to act as a responsible guarantor for the token money by
ensuring high degree of security which will prevent others from making fake currencies.

Q Source: Additional Research: CSP 2006

https://www.coin-competition.eu/history/the-coins-of-muhammad-bin-tughlaq/

67 With respect to Atal Bhujal Yojana (ABHY), consider the following statements.
1. It is a central sector scheme.
2. It is to be implemented with the financial assistance of the World Bank.
3. It provides funds to selected states for strengthening the institutions responsible for ground water
governance, as well as for encouraging community involvement for the same.

Select the correct answer using the codes below.


A. 2 only
B. 1 and 2 only
C. 1, 2 and 3
D. 3 only

53
Total Marks : 200
Online Prelims TEST - 29 (TEXTBOOK)
( InsightsIAS Mock Test Series for UPSC Preliminary Exam 2020 ) Mark Scored : 0

Correct Answer : C

Answer Justification :

Justification: The World Bank has approved Atal Bhujal Yojana (ABHY), a Rs.6000 crore Central
Sector Scheme of the Ministry of Water Resources, River Development and Ganga Rejuvenation.
The scheme is to be implemented over a period of five years from 2018-19 to 2022-23, with World
Bank assistance.

Atal Bhujal Yojana has been formulated by the Ministry to address the criticality of ground water
resources in a major part of the country. The scheme aims to improve ground water management in
priority areas in the country through community participation. The priority areas identified under
the scheme fall in the states of Gujarat, Haryana, Karnataka, Madhya Pradesh, Maharashtra,
Rajasthan and Uttar Pradesh.

These States represent about 25% of the total number of overexploited, critical and semi-critical
blocks in terms of ground water in India. They also cover two major types of groundwater systems
found in India - alluvial and hard rock aquifers- and have varying degrees of institutional readiness
and experience in groundwater management.

Funds under the scheme will be provided to the states for strengthening the institutions responsible
for ground water governance, as well as for encouraging community involvement for improving
ground water management to foster behavioural changes that promote conservation and efficient
use of water.

Q Source: http://jalshakti-dowr.gov.in/sites/default/files/ABHY.pdf

68 Consider the following statements.


1. Arsenic is a naturally occurring trace element found in rocks and soils.
2. Apart from West Bengal, ground water Arsenic contamination ‘hot spots’ in India has been mainly
identified in North-eastern and Southern states of India.
3. Arsenic contamination has not been found in volcanic rocks or alluvial formations in Northern India.

Select the correct answer using the codes below.


A. 1 only
B. 1, 2 and 3
C. 2 and 3 only
D. 1 and 2 only

Correct Answer : A

Answer Justification :

Justification: Arsenic is a naturally occurring trace element found in rocks, soils and the water in
contact with them. Arsenic has been recognized as a toxic element and is considered a human
health hazard. The occurrence of Arsenic in ground water was first reported in 1980 in West Bengal

54
Total Marks : 200
Online Prelims TEST - 29 (TEXTBOOK)
( InsightsIAS Mock Test Series for UPSC Preliminary Exam 2020 ) Mark Scored : 0

in India. In West Bengal, 79 blocks in 8 districts have Arsenic beyond the permissible limit of 0.05
mg/l.

Apart from West Bengal, Arsenic contamination in ground water has been found in the states of
Aasam, Bihar, Chattisgarh, Haryana, Jharkhand, KarnatakaPunjab, Uttar Pradesh, West Bengal.

S3: Southern states, except Telangana, are largely free from Arsenic contamination.

The occurrence of Arsenic in the states of Bihar, West Bengal and Uttar Pradesh is in alluvial
formations but in the state of Chhattisgarh, it is in the volcanic rocks exclusively confined to N-S
trending DongargarhKotri ancient rift zone. It has also been reported in Golaghat, Jorhat,
Lakhimpur, Nagaon, Nalbari, Sibsagar, Sonitpur district of Assam.

The most affected areas are on the eastern side of Bhagirathi River in the districts of Malda,
Murshidabad, Nadia, North 24 Parganas and South 24 Parganas and western side of the districts of
Howrah, Hugli and Bardhman. The occurrence of Arsenic in ground water is mainly in the aquifers
up to 100 m depth. The deeper aquifers are free from Arsenic contamination.

Q Source: http://jalshakti-dowr.gov.in/sites/default/files/ARSENIC.pdf

69 The objective of Energy Efficiency Financing Platform (EEFP) is to


1. Cross-subsidize and transfer energy efficiency certificates of one State DISCOM to another based
on their progress in cutting AT&C losses
2. Provide a platform to interact with financial institutions and project developers for implementation
of energy efficiency projects

Which of the above is/are correct?


A. 1 only
B. 2 only
C. Both 1 and 2
D. None

Correct Answer : B

Answer Justification :

Justification: It is an initiative of the Bureau of Energy Efficiency (BEE).

The objective of EEFP is to provide a platform to interact with financial institutions and project
developers for implementation of energy efficiency projects.

Under this programme, MoUs have been signed with financial institutions to work together for the
development of energy efficiency market and for the identification of issues related to this market
development.

For e.g. MoUs with M/s. PTC India Ltd, M/s. SIDBI, HSBC Bank, Tata Capital and IFCI Ltd have

55
Total Marks : 200
Online Prelims TEST - 29 (TEXTBOOK)
( InsightsIAS Mock Test Series for UPSC Preliminary Exam 2020 ) Mark Scored : 0

been signed by BEE to promote financing for energy efficiency projects.

Main objective of these MoUs is to promote lending in the areas of performance contracting, energy
efficiency in commercial sector, industrial complexes, power plants etc.

Q Source: https://beeindia.gov.in/content/eefp

70 Pradhan Mantri Innovative Learning Programme - 'DHRUV' has been started by Government of India
to
1. identify and encourage talented children to enrich their skills and knowledge
2. digitize school education to make it accessible to the differently abled

Select the correct answer using the codes below.


A. 1 only
B. 2 only
C. Both 1 and 2
D. None

Correct Answer : A

Answer Justification :

Justification: The Pradhan Mantri Innovative Learning Programme: DHRUV has been started by
Government of India to identify and encourage talented children to enrich their skills and
knowledge. In centres of excellence across the country, gifted children will be mentored and
nurtured by renowned experts in different areas, so that they can reach their full potential.

The program aims to cover two areas namely Science and Arts.
The program is to be launched from Indian Space Research Organisation (ISRO).
Around 60 students are selected broadly from Class 9 to Class 12 all over the country.

Learning: FIRST BATCH OF DHRUV PROGRAMME

The first batch of DHRUV programme was implemented during October 2019.

60 outstandingly talented students were selected in the first batch of DHRUV programme. To
begin with, two areas i.e., Science and Performing Arts were covered. There were 60 students
in all, 30 from each area. The 60 students came from across the country. The students have
been broadly chosen from classes 9 to 12, from all schools including government and private.
A 14 days programme was organized for these students in which the science and performing
arts students were separated into two groups. The science students were further divided into
3 groups of 10 each and also the performing arts students into 3 groups of 10 each.

Each group from science stream were required to produce a project under the mentorship of
experts from the field of science. Similarly, each group in performing arts were mentored by icons
from the field of culture and were required to choreograph a programme. All the six teams were
given themes relevant to issues being faced globally like environment change, pollution, terrorism,

56
Total Marks : 200
Online Prelims TEST - 29 (TEXTBOOK)
( InsightsIAS Mock Test Series for UPSC Preliminary Exam 2020 ) Mark Scored : 0

etc.

Q Source: https://repository.seshagun.nic.in/dhruv/

71 Consider the following statements.


1. Definitions of Micro, Small & Medium Enterprises are in accordance with the provision of Micro,
Small & Medium Enterprises Development (MSMED) Act.
2. MSMEs in Manufacturing sector are defined in terms of investment in Plant & Machinery, whereas
those in the Services sector are defined based on working capital & manpower.

Select the correct answer using the codes below.


A. 1 only
B. 2 only
C. Both 1 and 2
D. None of the above

Correct Answer : A

Answer Justification :

Justification: Definitions of Micro, Small & Medium Enterprises In accordance with the provision
of Micro, Small & Medium Enterprises Development (MSMED) Act, 2006 the Micro, Small and
Medium Enterprises (MSME) are classified in two Classes:

1. Manufacturing Enterprises-he enterprises engaged in the manufacture or production of goods


pertaining to any industry specified in the first schedule to the industries (Development and
regulation) Act, 1951) or employing plant and machinery in the process of value addition to the final
product having a distinct name or character or use. The Manufacturing Enterprise are defined in
terms of investment in Plant & Machinery.

2. Service Enterprises:-The enterprises engaged in providing or rendering of services and are


defined in terms of investment in equipment.

Manufacturing and services sector definitions

Micro Enterprises: Does not exceed twenty five lakh rupees (ten lakh for services)
Small Enterprises: More than twenty five lakh rupees but does not exceed five crore rupees
(More than ten lakh rupees but does not exceed two crore rupees for services)
Medium Enterprises: More than five crore rupees but does not exceed ten crore rupees (
More than two crore rupees but does not exceed five crore rupees for services)

Q Source: https://msme.gov.in/know-about-msme

72 The Ocean/Sea that has the most coral species is?

57
Total Marks : 200
Online Prelims TEST - 29 (TEXTBOOK)
( InsightsIAS Mock Test Series for UPSC Preliminary Exam 2020 ) Mark Scored : 0

A. Mediterranean Sea
B. Atlantic Ocean
C. Indian Ocean
D. Pacific Ocean

Correct Answer : D

Answer Justification :

Learning: Reef-building corals are restricted in their geographic distribution by factors such as the
temperature and the salinity (salt content) of the water. The water must also be clear to permit high
light penetration.

Because of these environmental restrictions, reefs generally are confined to tropical and
semitropical waters. The diversity of reef corals (the number of species), decreases in higher
latitudes up to about 30° north and south, beyond which reef corals are usually not found.

Generally, there are about twice as many coral species in Pacific Ocean reefs as in Atlantic Ocean
reefs.

Q Source: https://oceanservice.noaa.gov/facts/most_coral.html

73 Public Procurement is one of the top priorities of the present Government. In this context what are
the benefits of Government e-Marketplace (GeM)
⌰〰〰
1. GeM considerably reduces human interface in order placement and payment processing for the
Government departments thus cutting down corrupt practices.
2. All the documents on GeM are e-Signed at various stages by the buyers and sellers that makes it
secure and transparent.

Which of the above is/are correct?


A. 1 only
B. 2 only
C. Both 1 and 2
D. None

Correct Answer : C

Answer Justification :

Justification: GeM provides an end-to-end online Marketplace for Central and State Government
Ministries / Departments to the Government, Sellers and the Indian industry and economy

GeM is a completely paperless, cashless and system driven e-market place that enables
procurement of common use goods and services with minimal human interface.
It is a very bold step of the Government with the aim to transform the way in which

58
Total Marks : 200
Online Prelims TEST - 29 (TEXTBOOK)
( InsightsIAS Mock Test Series for UPSC Preliminary Exam 2020 ) Mark Scored : 0

procurement of goods and services is done by the Government Ministries/Departments, PSUs,


autonomous bodies etc.

Benefits are:

Transparency: GeM eliminates human interface in vendor registration, order placement and
payment processing, to a great extent. Being an open platform, GeM offers no entry barriers to
bonafide suppliers who wish to do business with the Government.

Efficiency: Direct purchase on GeM can be done in a matter of minutes and the entire process in
online, end to end integrated and with online tools for assessing price reasonability.

Secure and safe: GeM is a completely secure platform and all the documents on GeM are e-Signed
at various stages by the buyers and sellers.

Potential to support Make in India: On GeM, the filters for selecting goods which are Preferential
Market Access (PMA) compliant and those manufactured by Small Scale Industries(SSI), enables
the Government buyers to procure Make in India and SSI goods very easily.

Savings to the Government: The transparency, efficiency and ease of use of the GeM portal has
resulted in a substantial reduction in prices on GeM, in comparison to the tender, Rate Contract and
direct purchase rates.

Q Source: https://gem.gov.in/aboutus

74 With respect to Minamata Convention, consider the following


1. It is a global treaty to protect human health and the environment from adverse effects of mercury.
2. After joining the Convention, it will be easier for a nation to get technological or financial assistance
to address Mercury related issues.

Which of the above is/are correct?


A. 1 only
B. 2 only
C. Both 1 and 2
D. None

Correct Answer : C

Answer Justification :

Justification: Statement 1: The Minamata Convention on Mercury is a multilateral environmental


agreement that addresses specific human activities which are contributing to widespread mercury
pollution and eventually reduce its harmful effects.

Statement 2: These are the obligations on Parties of Convention:

Ban on new mercury mine and the phase-out of existing ones


Phase out and phase down of mercury use in a number of products and processes

59
Total Marks : 200
Online Prelims TEST - 29 (TEXTBOOK)
( InsightsIAS Mock Test Series for UPSC Preliminary Exam 2020 ) Mark Scored : 0

Control measures on emissions to air and on releases to land and water


Regulation of the informal sector of artisanal and small-scale gold mining etc.

In return, parties get enhanced assistance to tackle these issues.

Q Source: Based on past year UPSC papers

75 Which of the following is/are correct with reference to the views of Swami Vivekananda?

A. He championed the supremacy of Vedantic philosophy


B. He did not favour the concept of Indian ‘nationalism’.
C. He resisted the idea of idol worship and deitifying religious figures.
D. None of the above.

Correct Answer : A

Answer Justification :

Justification: Option A: He was a key figure in the introduction of the Indian philosophies of
Vedanta and Yoga to the Western world and is credited with raising interfaith awareness, bringing
Hinduism to the status of a major world religion during the late 19th century.

A disciple of Ramakrishna Paramahamsa, he championed the supremacy of Vedantic philosophy. His


talk at the Chicago (USA) Conference of World Religions in 1893 made the westerners realize the
importance of Hinduism.

Option B: He was a major force in the revival of Hinduism in India, and contributed to the concept
of nationalism in colonial India.

Option C: He was a devotee of Hindu Gods and Goddesses and himself used to meditate in the
presence of their idols.

Q Source: Revision: Freedom Movement

76 Enclosed by the Aravali Hills in the North, it is an important artificial lake and a large bird site with
the famous Jal Mahal being situated in the middle of the lake. It is

A. Man Sagar Lake


B. Bhojtal Lake
C. Pichola Lake
D. Kanwar Lake

Correct Answer : A

Answer Justification :

60
Total Marks : 200
Online Prelims TEST - 29 (TEXTBOOK)
( InsightsIAS Mock Test Series for UPSC Preliminary Exam 2020 ) Mark Scored : 0

Learning: It has a water spread area of 300 acres and is enclosed by the Aravalli hills on the north,
west and eastern sides, while the southern side consists of plains that are intensely inhabited.

Throughout an year, several hundred species of birds have been recorded at the Birding Fair site,
making it as a great attraction to bird watchers.

As the lake is being restored and islands have been created for bird nesting, more species in bigger
number are expected to settle here.

Birding festivals are held at the lake.

Q Source: As mentioned above

77 For an emissions reduction project generating carbon credits, being ‘additional’ means
1. That the project has zero social marginal cost
2. That the project is funded entirely by external support, which could include support from
environmental agencies, NGOs and international donor agencies.

Select the correct answer using the codes below.


A. 1 only
B. 2 only
C. Both 1 and 2
D. None of the above

Correct Answer : D

Answer Justification :

Justification: Zero social marginal cost means that production of an extra unit of that good does
not cost the society anything (here in terms of carbon units), but this is not the case with such a
project. The project is more likely to have a negative SMC if it is entirely renewable.

Additionality is a term that describes renewable energy generation that is truly new – i.e. additional.
For example, companies responsible for financially supporting new, expanding, or developing
renewable generation sources, as opposed to buying into what is already available or planned, can
claim additionality. These projects have a material impact on displacing global emissions by
reducing conventional fossil sources of generation on the grid.

Therefore, being ‘additional’ means that the project activity would not have existed in the absence
of carbon market incentives and that the project reduces emissions and/or physically removes
carbon from the atmosphere beyond the business-as-usual scenario. Additionality is a core
requirement of all projects that produce high quality carbon credits.

In some cases, additionality can also be conveyed through other financial metrics. For example, in
a region with high priced or highly demanded environmental commodities such as renewable
energy certificates (RECs), purchase of these commodities may be considered additional. The same
61
Total Marks : 200
Online Prelims TEST - 29 (TEXTBOOK)
( InsightsIAS Mock Test Series for UPSC Preliminary Exam 2020 ) Mark Scored : 0

may be true in emerging international markets where demand for environmental commodities
creates a powerful signal to the market of the value of renewable energy development and provides
an instrumental source of revenue to new projects.

Being able to state additionality emphasizes a company’s commitment to advancing carbon


reductions beyond business as usual.

Q Source:
https://c402277.ssl.cf1.rackcdn.com/publications/1310/files/original/WWF_position_and_guidance_o
n_corporate_use_of_voluntary_carbon_credits_EXTERNAL_VERSION_11_October_2019.pdf?158384
6796

78 Which of these North-eastern states does NOT border Bangladesh?

A. Manipur
B. Tripura
C. Assam
D. Meghalaya

Correct Answer : A

Answer Justification :

Bangladesh and India share a 4,156-kilometre-long (2,582 mi) international border, the fifth-longest
land border in the world, including 262 km (163 mi) in Assam, 856 km (532 mi) in Tripura, 180 km
(110 mi) in Mizoram, 443 km (275 mi) in Meghalaya, and 2,217 km (1,378 mi) in West Bengal. It
does not share border with Manipur.

Learning:

62
Total Marks : 200
Online Prelims TEST - 29 (TEXTBOOK)
( InsightsIAS Mock Test Series for UPSC Preliminary Exam 2020 ) Mark Scored : 0

Q Source: Revision: Map of India

79 Consider the following statements about Global Food & Trade Network (GFTN).
1. GFTN is one of WWF’s initiatives to combat illegal logging.
2. GFTN helps improve the management of global production forests by using the influence and
purchasing power of businesses to bring about market change.

Select the correct answer using the codes below.


A. 1 only
B. 2 only
C. Both 1 and 2
D. None of the above

Correct Answer : C

Answer Justification :

Justification: The business community can play a vital role in ensuring forest resources are well-
managed. Sustainable forest management can help redress some of the most severe problems
affecting forests, such as deforestation.

WWF’s Global Forest & Trade Network (GFTN) can help. It works to improve the management of
global production forests by using the commitments, influence, and purchasing power of businesses
to bring about market change.

Its goal is to create a thriving, mainstream market for environmentally and socially responsible
forest products, and GFTN is one of WWF’s leading initiatives to combat illegal logging and drive
improvements in forest management.

The GFTN links more than 100 companies, communities and governments in over 15 countries to
create market incentives for responsible forestry and trade practices. The initiative was established
in 1991 and is the world’s longest-running and largest forest and trade programme of its kind.

Since its creation, GFTN's efforts have helped to ensure that millions of hectares of forests are
independently and credibly certified, a guarantee that the forests are well managed and that their
products come from legal and sustainable timber harvests.

Q Source: http://gftn.panda.org/

https://www.worldwildlife.org/initiatives/global-forest-trade-network

80 Objectives of ‘Ek Bharat Shreshtha Bharat’ initiative include


1. To promote the spirit of national integration through a deep and structured engagement between all
Indian States and Union Territories through a year-long planned engagement between States
2. To create an environment which promotes learning between States by sharing best practices and
experiences

63
Total Marks : 200
Online Prelims TEST - 29 (TEXTBOOK)
( InsightsIAS Mock Test Series for UPSC Preliminary Exam 2020 ) Mark Scored : 0

3. To allow more developed districts in certain states to step forward to mentor and adopt a less
developed district in another state

Select the correct answer using the codes below.


A. 1 and 2 only
B. 2 and 3 only
C. 1 and 3 only
D. 1, 2 and 3

Correct Answer : A

Answer Justification :

Justification: The broad objectives of the initiative are as follows:

i. To CELEBRATE the Unity in Diversity of our Nation and to maintain and strengthen the fabric of
traditionally existing emotional bonds between the people of our Country;

ii. To PROMOTE the spirit of national integration through a deep and structured engagement
between all Indian States and Union Territories through a year-long planned engagement between
States;

iii. To SHOWCASE the rich heritage and culture, customs and traditions of either State for enabling
people to understand and appreciate the diversity that is India, thus fostering a sense of common
identity

iv. TO ESTABLISH long-term engagements and,

v. TO CREATE an environment which promotes learning between States by sharing best practices
and experiences.

Key Themes for Interaction:

To celebrate the idea of India as a nation wherein different cultural units across varied
geographies coalesce and interact with each other, this glorious manifestation of diverse
cuisine, music, dance, theatre, movies & films, handicrafts, sports, literature, festivals,
painting, sculpture etc. will enable people to imbibe the innate chord of binding and
brotherhood.
To make our people aware about the seamless integral hull of the Modern Indian State spread
across a vast landmass on whose firm foundations, the geo-political strength of the country is
ensured to benefit one and all.
To impress upon people at large about the increasing inter-connectedness between the
constituents of various cultures and traditions, which is so vital for the spirit of nation
building.
To induce a sense of responsibility & ownership for the nation as a whole through these close
cross-cultural interactions as it intends to build up the inter-dependence matrix unequivocally.

Q Source: https://seshagun.gov.in/ebsb

64
Total Marks : 200
Online Prelims TEST - 29 (TEXTBOOK)
( InsightsIAS Mock Test Series for UPSC Preliminary Exam 2020 ) Mark Scored : 0

81 The source of fossil fuels in the continental shelves comes from

A. Disconnection with the sea


B. Frequent volcanic eruptions
C. Massive sedimentary deposits received over a long time
D. Tectonic movements

Correct Answer : C

Answer Justification :

Learning: A continental shelf is the edge of a continent that lies under the ocean. It extends from
the coastline of a continent to a drop-off point called the shelf break. From the break, the shelf
descends toward the deep ocean floor in what is called the continental slope.

Even though they are underwater, continental shelves are part of the continent.

The continental shelves are covered with variable thicknesses of sediments brought down by rivers,
glaciers, wind, from the land and distributed by waves and currents. These become the source of
fossil fuels.

Q Source: 11th Fundamental Physical Geography NCERT

82 Consider the following statements.


1. Water being a State subject, initiatives on sustainable water management including conservation
and water harvesting in the Country is primarily States’ responsibility.
2. The Model Building Bye Laws, 2016, was passed by the Parliament for guidance of the States/UTs,
and it has a chapter on ‘Rainwater Harvesting’.
3. The implementation of the rainwater harvesting policy comes within the purview of the State
Government or Urban Local Bodies.

Select the correct answer using the codes below.


A. 2 only
B. 1, 2 and 3
C. 1 and 3 only
D. 1 and 2 only

Correct Answer : B

Answer Justification :

Justification: We are pasting excerpts from an important PIB release:

“Water being a State subject, initiatives on sustainable water management including conservation

65
Total Marks : 200
Online Prelims TEST - 29 (TEXTBOOK)
( InsightsIAS Mock Test Series for UPSC Preliminary Exam 2020 ) Mark Scored : 0

and water harvesting in the Country is primarily States’ responsibility. However, the important
measures taken by the Central Government for conservation, management of ground water and
effective implementation of rain water harvesting in the country are at the following URL:

http://mowr.gov.in/sites/default/files/Steps_to_control_water_depletion_Jun2019.pdf

Further, a number of States have done notable work in the field of water conservation/harvesting.
Of these, mention can be made of ‘Mukhyamantri Jal Swavlamban Abhiyan’ in Rajasthan, ‘Jalyukt
Shibar’ in Maharashtra, ‘Sujalam Sufalam Abhiyan’ in Gujarat, ‘Mission Kakatiya’ in Telangana,
Neeru Chettu’ in Andhra Pradesh, Jal Jeevan Hariyali in Bihar, ‘Jal Hi Jeevan’ in Haryana among
others.

As per information received from Ministry of Housing & Urban Affairs, the Model Building Bye
Laws, 2016, has been issued for guidance of the States/UTs which has a chapter on ‘Rainwater
Harvesting’. The provisions of this chapter are applicable to all the buildings. 33 States / UTs have
adopted the rainwater harvesting provisions. The implementation of the rainwater harvesting policy
comes within the purview of the State Government/Urban Local Body / Urban Development
Authority.

In order to regulate the Over-exploitation and consequent depletion of ground water, the Ministry
has circulated a Model Bill to all the States/UTs to enable them to enact suitable ground water
legislation for regulation of its development, which includes provision of rain water harvesting. So
far, 15 States/UTs have adopted and implemented the ground water legislation on the lines of
Model Bill.”

Q Source: https://pib.gov.in/newsite/pmreleases.aspx?mincode=38 (Release ID :200129)

83 Which of the following factors aid the weathering of rocks?


1. Intense heating during the day and rapid cooling at night
2. Pore pressure of water seeping into rocks
3. Thawing of rocks

Select the correct answer using the codes below.


A. 1 only
B. 1 and 3 only
C. 2 and 3 only
D. 1, 2 and 3

Correct Answer : D

Answer Justification :

Justification: Block and Granular Disintegration type of weathering is carried out in deserts by
rapid changes of temperature, or in mountains through frost action.

Desert regions experience a large diurnal range of temperature due to direct heating by the
sun during the day and rapid radiation at night. The rocks successively expand and contract.

66
Total Marks : 200
Online Prelims TEST - 29 (TEXTBOOK)
( InsightsIAS Mock Test Series for UPSC Preliminary Exam 2020 ) Mark Scored : 0

The repeated expansion and contraction of the rocks tends to enlarge joints and large rock
masses ultimately break into angular blocks.
This process is called block disintegration. Rocks having vertical joints are more susceptible
to this form of weathering. The rocks having curvilinear joints get weathered.
If water seeps in the cracks of the rocks, it exerts a further pressure from inwards resulting in
cracking of the rock.

Q Source: Additional Research: Chapter 7: Goh Cheng Leong - Certificate Physical and Human
Geography

84 Match the following rivers to their state of origin.


1. Mahanadi : Chhattisgarh
2. Godavari: Andhra Pradesh
3. Cauvery: Kerala
4. Tapti: Madhya Pradesh

Select the correct answer using the codes below.


A. 1 and 4 only
B. 2 and 3 only
C. 1, 2 and 3 only
D. 1, 3 and 4 only

Correct Answer : A

Answer Justification :

Justification: Statement 1: Mahanadi is a combination of many mountain streams and thus its
precise source is impossible to pinpoint. However its farthest headwaters lie 6 kilometres from
Pharsiya village in Dhamtari district of Chhattisgarh.

Statement 2: The Godavari is India's second longest river after the Ganga. Its source is in
Triambakeshwar, Maharashtra.

Statement 3: Originating in the foothills of Western Ghats at Talakaveri, Kodagu in Karnataka


Cauvery flows generally south and east through Karnataka and Tamil Nadu emptying into the Bay of
Bengal.

Statement 4: The Tapti River between the Godavari and Narmada rivers starts from Betul, Madhya
Pradesh, and then flows westwards before draining through the Gulf of Khambhat into the Arabian
Sea.

Q Source: 11th NCERT: India Physical Geography

85 Consider the following statements.

67
Total Marks : 200
Online Prelims TEST - 29 (TEXTBOOK)
( InsightsIAS Mock Test Series for UPSC Preliminary Exam 2020 ) Mark Scored : 0

1. The repo rate is the rate at which Reserve Bank of India borrow from other banks.
2. In the event of inflation, central banks reduce the repo rate as this reduces liquidity in the system.

Which of the above is/are correct?


A. 1 only
B. 2 only
C. Both 1 and 2
D. None of the above

Correct Answer : D

Answer Justification :

Learning: Repo rate is the rate at which the central bank of a country (Reserve Bank of India in
case of India) lends money to commercial banks in the event of any shortfall of funds. Repo rate is
used by monetary authorities to control inflation.

In the event of inflation, central banks increase repo rate as this acts as a disincentive for banks to
borrow from the central bank. This ultimately reduces the money supply in the economy and thus
helps in arresting inflation.

The central bank takes the contrary position in the event of a fall in inflationary pressures. Repo
and reverse repo rates form a part of the liquidity adjustment facility.

Q Source: Based on CSP papers

86 Action for Climate Empowerment (ACE) is a term adopted in which of following important climate
change mitigation treaties/protocols/conventions?

A. Montreal Protocol
B. Convention on Biological Diversity
C. UNFCCC
D. Paris convention 2015

Correct Answer : C

Answer Justification :

Learning: It is a part of UNFCCC and refers to Article 6 of the Convention’s original text (1992),
focusing on six priority areas: education, training, public awareness, public participation, public
access to information, and international cooperation on these issues.

The implementation of all six areas has been identified as the pivotal factor for everyone to
understand and participate in solving the complex challenges presented by climate change.

ACE calls on governments to develop and implement educational and public awareness

68
Total Marks : 200
Online Prelims TEST - 29 (TEXTBOOK)
( InsightsIAS Mock Test Series for UPSC Preliminary Exam 2020 ) Mark Scored : 0

programmes, train scientific, technical and managerial personnel, foster access to information, and
promote public participation in addressing climate change and its effects.

It also urges countries to cooperate in this process, by exchanging good practices and lessons
learned, and strengthening national institutions.

Q Source: AR: Chapter 19: Indian Economy: Ramesh Singh

87 The main objectives of the Revamped Scheme of Fund for Regeneration of Traditional Industries
(SFURTI) include
1. To provide sustained employment for traditional industry artisans and rural entrepreneurs
2. To organize the traditional industries and artisans into clusters in order to make them, competitive
and provide support for their long term sustainability
3. To equip traditional artisans of the associated clusters with the improved skills and capabilities
through training and exposure visits
4. To enhance marketability of products of such clusters by providing support for new products,
design intervention and improved packaging and also the improvement of marketing Infrastructure

Select the correct answer using the codes below.


A. 1, 2, 3 and 4
B. 2, 3 and 4 only
C. 3 and 4 only
D. 1 and 2 only

Correct Answer : A

Answer Justification :

Justification: Other objectives are:

To make provision for common facilities and improved tools and equipments for artisans;
To strengthen the cluster governance systems with the active participation of the
stakeholders, so that they are able to gauge the emerging challenges and opportunities and
respond to them in a coherent manner;
To build up innovative and traditional skills, improved technologies, advanced processes,
market intelligence and new models of public-private partnerships, so as to gradually
replicate similar models of cluster- based regenerated traditional industries.

The Scheme would cover three types of interventions namely:

'Soft Interventions', 'Hard Interventions' and 'Thematic Interventions': Soft Interventions: A


maximum ceiling of ₹ 25.00 lakhs (100% scheme funding);
Hard Interventions: As per project requirement (75% scheme funding)
Eligibility for assistance: Non-Government Organizations (NGOs), Institutions of the Central
and State Governments and, Semi-Government institutions, field functionaries of State and
Central Govt., Panchayati Raj institutions (PRIs), and similar agencies, with suitable expertise
to undertake cluster development.

69
Total Marks : 200
Online Prelims TEST - 29 (TEXTBOOK)
( InsightsIAS Mock Test Series for UPSC Preliminary Exam 2020 ) Mark Scored : 0

Q Source: https://msme.gov.in/development-khadi-village-and-coir-industries

88 WWF scientists have helped develop a range of software tools for integrating the value of ecosystem
services and biodiversity in different decision contexts. Consider the following about them.
1. InVEST (Integrated Valuation of Ecosystem Services and Tradeoffs) is a software tool that models
and maps the delivery, distribution, and economic value of ecosystem services and biodiversity.
2. The Natural Capital Project has developed tools for quantifying ecosystem services in watershed
management investment decisions.

Select the correct answer using the codes below.


A. 1 only
B. 2 only
C. Both 1 and 2
D. None of the above

Correct Answer : C

Answer Justification :

Justification: Statement 1: InVEST (Integrated Valuation of Ecosystem Services and Tradeoffs) is


a unique software tool that models and maps the delivery, distribution, and economic value of
ecosystem services and biodiversity. InVEST helps decision makers visualize the impacts of
decisions and identify tradeoffs and compatibilities between environmental, economic, and social
benefits. InVEST is now in its own software platform, free of charge, and continually updated, with
support through an online user forum.

The InVEST software includes a Scenario Generator developed by WWF scientists for developing
scenario maps to compare how ecosystem services are affected under different possible futures.

Statement 2: Centered at Stanford University, the Natural Capital Project is a partnership among
WWF, The Nature Conservancy, Chinese Academy of Sciences, University of Minnesota, and
Stockholm Resilience Centre. Through pioneering science, cutting-edge technology, and
collaborative partnerships worldwide, the Natural Capital Project works to integrate the value
nature provides to people into all major decisions.

An essential element of the Natural Capital Project is developing tools that help decision makers
protect biodiversity and ecosystem services.

The Natural Capital Project holds regular events to train people in its tools and approaches for
integrating ecosystem service information into decision making. WWF scientists have helped
develop the Natural Capital Project’s free online course on natural capital.

Q Source: https://www.worldwildlife.org/projects/the-natural-capital-project

89 In recent years, the World Economic Forum has consistently ranked water crises as a top global
concern. In this respect, the AgWater challenge of Ceres and WWF focuses on

70
Total Marks : 200
Online Prelims TEST - 29 (TEXTBOOK)
( InsightsIAS Mock Test Series for UPSC Preliminary Exam 2020 ) Mark Scored : 0

A. Food and beverage companies


B. Small and marginal farms in developing countries
C. Desert ecosystems
D. Protected areas

Correct Answer : A

Answer Justification :

Justification: Population growth, pollution and climate change are all increasingly threatening
water availability and water quality. With the global food sector using 70 percent of the world’s
freshwater supply, food and beverage companies can and must play a powerful role in protecting
water quality and quantity.

Ceres and WWF created the AgWater Challenge to advance water stewardship among the world’s
most influential food and beverage companies.

The AgWater Challenge helps companies advance their sustainable sourcing strategies.
Participating companies receive support in analyzing water issues within their supply chains, and in
refining or making new sourcing commitments that enable them to better address their risk. Ceres
and WWF work with participating companies to develop time-bound and measurable commitments
to:

• Reduce the water impacts associated with key agricultural commodities

• Implement locally-relevant strategies to mitigate risk in agricultural areas where water is scarce

• Support and incentivize farmers and other agricultural producers to strengthen water
stewardship practices

In turn, participating companies benefit from:

• Technical assistance from leading NGOs

• Peer-to-peer learning on best practices for managing water risks and challenges with meaningful
goal setting

• Opportunities to earn recognition as an AgWater Challenge participant for responding to water


challenges, through new actions and commitments

• Improved standing in Ceres’ next Feeding Ourselves Thirsty ranking

• Insight from communications experts on how to effectively tell their sustainability story to a range
of key stakeholders

Q Source: https://www.worldwildlife.org/projects/the-agwater-challenge

90 Consider the following statements.

71
Total Marks : 200
Online Prelims TEST - 29 (TEXTBOOK)
( InsightsIAS Mock Test Series for UPSC Preliminary Exam 2020 ) Mark Scored : 0

1. Pancheshwar Multipurpose Project (PMP) is a bi-national Multi Purpose Project, primarily aimed at
energy production and augmenting irrigation in India and Nepal.
2. Pancheshwar Main Dam is proposed on river Mahakali (known as Sarada in India), where the river
forms the international boundary between Nepal and the State of Sikkim in India.

Select the correct answer using the codes below.


A. 1 only
B. 2 only
C. Both 1 and 2
D. None of the above

Correct Answer : A

Answer Justification :

Justification: Statement 1: Pancheshwar Multipurpose Project (PMP) is a bi-national hydropower


project to be developed in Mahakali River bordering Nepal and India. Development of PMP, is a
mutual interest project between two countries, and is covered under integrated Mahakali Treaty
signed between Nepal and India according to which, equal sizes of underground power house i.e. of
3240MV will be constructed on each side of Mahakali river in India and Nepal. Moreover, the PMP
also offer benefit of regulated water for irrigation to a vast area of agricultural land both in Nepal
and India along with benefit of flood control at downstream.

The constitution of Pancheswar Development Authority has already been notified.

Statement 2: Pancheshwar Main Dam is proposed on river Mahakali (known as river Sarada in
India), where the river forms the international boundary between the Far Western Development
Region of Nepal and State of Uttarakhand in India. The dam site is around 2.5 km downstream of
the confluence of river Sarju with river Mahakali.

Q Source: http://www.pmp.gov.np/about-project.php

http://mowr.gov.in/international-cooperation/bilateral-cooperation-with-neighbouring-countries/indi
a-nepal-cooperation

91 According to Census 2011, which one of the following Indian States/UT has the least population
density in India?

A. Arunachal Pradesh
B. Sikkim
C. Goa
D. Ladakh

Correct Answer : D

Answer Justification :

72
Total Marks : 200
Online Prelims TEST - 29 (TEXTBOOK)
( InsightsIAS Mock Test Series for UPSC Preliminary Exam 2020 ) Mark Scored : 0

Learning:

Q Source: Census data

92 With reference to Indian National Satellite System (INSAT), consider the following statements.
1. It is useful in Meteorology because it provides access to real time weather data.
2. It can be used for Search and rescue services/operations because it provides location information of
the distress signals transmitted by devices.

Which of the above is/are correct?


A. 1 only
B. 2 only
C. Both 1 and 2
D. None

Correct Answer : C

Answer Justification :

Justification: Statement 1: The services of both INSAT and IRS are being taken by the IMD for
meterological purposes.

For e.g. KALPANA-1 carries a Very High Resolution Radiometer (VHRR) capable of imaging the
Earth in the visible, thermal infrared and water vapor bands.

Statement 2: INSAT helps provides location information of the distress signals transmitted by
beacons mounted on board ships, aircraft or those carried by individuals.

The location information is determined instantaneously and informed to rescue coordination centres
for carrying out search and rescue operations of the affected ship, aircraft or individuals.

73
Total Marks : 200
Online Prelims TEST - 29 (TEXTBOOK)
( InsightsIAS Mock Test Series for UPSC Preliminary Exam 2020 ) Mark Scored : 0

Transponders carried on the satellites of INSAT enable telecommunication and broadcasting.

Q Source: https://www.isro.gov.in/applications/meteorology

93 Damanganga-Pinjal Link Project and Par-Tapi-Narmada Link Project are twin inter-river water links
concerning the states of

A. Maharashtra and Gujarat


B. Maharashtra and Madhya Pradesh
C. Gujarat and Madhya Pradesh
D. Uttar Pradesh and Maharashtra

Correct Answer : A

Answer Justification :

Justification: Damanganga-Pinjal Link Project and Par-Tapi-Narmada Link Project are twin links
concerning Maharashtra and Gujarat. Damanganga-Pinjal Link Project benefits Maharashtra while
Par-Tapi-Narmada Link Project benefits Gujarat. The Detailed Project Reports of both links are
ready. Central Water Commission has completed techno-economic appraisal of Damanganga-Pinjal
Link Project while the techno-economic appraisal of Par-Tapi-Narmada Link Project is in advance
stage.

Learning: Interlinking of River (ILR) programme is of national importance and has been taken up
on high Priority. Hon’ble Minister for Water Resources, RD & GR is monitoring the progress of ILR
from time to time. The mission of this programme is to ensure greater equity in the distribution of
water by enhancing the availability of water in drought prone and rain-fed area.

Under the National Perspective Plan (NPP) prepared by Ministry of Water Resources, NWDA has
already identified 14 links under Himalayan Rivers Component and 16 links under Peninsular Rivers
Component for inter basin transfer of water based on field surveys and investigation and detailed
studies.

Mahanadi Godavari link is the first and critical link of nine link system of Mahanadi-Godavari-
Krishna-Pennar-Cauvery-Vaigai-Gundar under Peninsular Component of NPP.
Pending consensus on Mahanadi-Godavari and Godavari(Inchampalli)-Krishna link projects
due to large submergence involved, alternative studies have been carried out to divert
unutilized water share of Chhattisgarh State in Indravati sub-basin in Godavari basin (as per
GWDT award) to Cauvery through Godavari-Cauvery link project.

Q Source: http://mowr.gov.in/schemes-projects-programmes/schemes/interlinking-rivers

94 The World Wildlife Fund (WWF) is helping accelerate climate action from national governments, and
from cities, states and businesses to deliver on the ambitious temperature goals of the historic Paris
Agreement by
1. Providing financial support to developing countries as an implementing partner of the Green
Climate Fund (GCF) and the Global Environment Facility (GEF)

74
Total Marks : 200
Online Prelims TEST - 29 (TEXTBOOK)
( InsightsIAS Mock Test Series for UPSC Preliminary Exam 2020 ) Mark Scored : 0

2. Ensuring that ecosystem-based approaches to adaptation are incorporated into national


development plans
3. Helping curb climate pollution from international aviation

Select the correct answer using the codes below.


A. 1, 2 and 3
B. 2 only
C. 1 and 3 only
D. 3 only

Correct Answer : A

Answer Justification :

Justification: The Paris Agreement is the world’s collective response to addressing climate change
in the years to come.

WWF is helping the historic Paris Agreement by:

Providing financial support to developing countries as an implementing partner of the Green


Climate Fund (GCF) and the Global Environment Facility (GEF)
Building a coalition of non-federal actors in the US who remained committed to the Paris
Agreement
Helping transition developing countries to clean energy sources like wind and solar
Working with companies and cities to switch to 100% renewable energy and harnessing their
purchasing power to drive bigger change
Ensuring that ecosystem-based approaches to adaptation are incorporated into national
development plans
Collaborating with US Latino leaders and communities to raise awareness of climate risks
from Latin America to North America
Curbing climate pollution from international aviation

Q Source: https://www.worldwildlife.org/initiatives/climate

95 What reasons make the study of Solar corona important?


1. Solar flares can damage satellites and impose an enormous financial cost on space programmes.
2. The corona has the lowest temperature in the Sun making scientific observations possible.
3. Very large flares can generate currents within electricity grids on earth and adversely affect energy
supplies.

Select the correct answer using the codes below.


A. 1 and 2 only
B. 1 and 3 only
C. 3 only
D. 1, 2 and 3

75
Total Marks : 200
Online Prelims TEST - 29 (TEXTBOOK)
( InsightsIAS Mock Test Series for UPSC Preliminary Exam 2020 ) Mark Scored : 0

Correct Answer : B

Answer Justification :

Justification: Statement 2: The corona is in the outer layer of the sun’s atmosphere—far from its
surface. Yet the corona is hundreds of times hotter than the sun’s surface.

However, the corona is very dim because the corona is about 10 million times less dense than the
sun’s surface.

This low density makes the corona much less bright than the surface of the sun.

The corona is usually hidden by the bright light of the sun's surface. That makes it difficult to see
without using special instruments. However, the corona can be seen during a total solar eclipse.

Statement 1: Parker Solar Probe by NASA will study the outer atmosphere of the sun, known as the
corona. Parker Solar Probe will travel through the sun’s atmosphere, closer to the surface than any
spacecraft before it.

To measure the corona, the probe will reach an orbit within four million miles (6.5 million km) of the
sun.

PSP is also aimed at collecting vital information about the life of stars and their weather events.
This will help scientists improve how we predict dangerous solar flares, which can disrupt satellites
and power supplies here on Earth.

Statement 3: The mission can help scientists to better understand solar flares – brief eruptions of
intense high-energy radiation from the sun’s surface that can knock out communications on Earth.

According to Nasa, observations from this new vantage point will also help to uncover the physics of
how stars work, and could improve our ability to predict space weather. These events have impacts
on Earth as well as the satellites and astronauts in space.

Q Source: https://spaceplace.nasa.gov/sun-corona/en/

96 The water body that holds 20% of the world's fresh surface water and is the world's largest
freshwater lake by volume is

A. Caspian Sea
B. Lake Baikal
C. Lake Superior
D. Victoria Lake

Correct Answer : B

Answer Justification :

Learning: Most of all inland salt water on earth is held in the Caspian Sea. But, about 20 percent of

76
Total Marks : 200
Online Prelims TEST - 29 (TEXTBOOK)
( InsightsIAS Mock Test Series for UPSC Preliminary Exam 2020 ) Mark Scored : 0

global freshwater is held in Lake Baikal (Russia) and the Great Lakes of North America.

Lake Baikal is an estimated 5,387 feet deep (1,642 meters), and its bottom is approximately 3,893
feet (1,187 meters) below sea level.

The salty Caspian Sea has the greatest surface area of any lake at 143,200 square miles (370,886
square kilometers).

Lake Superior, on the United States/Canada border, is the named freshwater lake with the greatest
surface area at 31,700 square miles (82,103 square kilometers).

Q Source: Chapter 9: Goh Cheng Leong - Certificate Physical and Human Geography

97 ‘Climate Action Reserve’ is a

A. Carbon credit methodology adopted by IUCN


B. Carbon offset registry for the North American carbon market
C. Greenhouse Gas (GHG) Protocol under the Kigali agreement
D. Mechanism under REDD+

Correct Answer : B

Answer Justification :

Justification: As the premier carbon offset registry for the North American carbon market, the
Climate Action Reserve encourages action to reduce greenhouse gas (GHG) emissions by ensuring
the environmental integrity and financial benefit of emissions reduction projects.

The Reserve establishes high quality standards for carbon offset projects, oversees independent
third-party verification bodies, issues carbon credits generated from such projects and tracks the
transaction of credits over time in a transparent, publicly-accessible system.

The Reserve has also conducted training for the government of South Korea, has engaged with and
supported the World Bank’s Partnership for Market Readiness, and is actively seeking opportunities
to support emerging carbon trading programs and markets.

The Reserve has expanded its regulatory-quality work in forestry standards and applied its
expertise internationally. For many years, it has played an active role in the development of REDD+
standards internationally through its partnerships with the World Bank Forest Carbon Partnership
Facility (FCPF) and the REDD Offset Working Group (ROW). Additionally, it has consulted on
forestry issues with national and subnational governments, including Acre, Brazil.

Q Source: https://www.climateactionreserve.org/about-us/

98 Consider the following statements with reference to Public Bills.


1. It does not require any prior notice before introduction in the house.
2. Introduction of public bills require prior Presidential assent.

77
Total Marks : 200
Online Prelims TEST - 29 (TEXTBOOK)
( InsightsIAS Mock Test Series for UPSC Preliminary Exam 2020 ) Mark Scored : 0

Which of the above is/are correct?


A. 1 only
B. 2 only
C. Both 1 and 2
D. None

Correct Answer : D

Answer Justification :

Justification: Public bills are introduced by Ministers, whereas private bills are introduced by any
Member of Parliament other than a minister.

Introduction of public bill in the House requires seven days’ notice. Introduction of private bill in
the House requires one month’s notice.

No prior Presidential assent in required.

Public bill reflects of the policies of the government (ruling party). Its rejection by the House
amounts to the expression of want of parliamentary confidence in the government and may lead to
its resignation.

Q Source: Revision: Chapter 22: Indian Polity: M Laxmikanth

99 The Gender Development Index (GDI) measures gender gaps in human development achievements by
accounting for disparities between women and men in three basic dimensions of human development,
which include
1. Health
2. Knowledge
3. Political status

Select the correct answer using the codes below.


A. 1 and 2 only
B. 1 only
C. 1, 2 and 3
D. 3 only

Correct Answer : A

Answer Justification :

Justification: It uses health, knowledge and living standards, the same component indicators as in
the HDI. It is a direct measure of gender gap showing the female HDI as a percentage of the male
HDI. This is what is included in the sub-indicators:

78
Total Marks : 200
Online Prelims TEST - 29 (TEXTBOOK)
( InsightsIAS Mock Test Series for UPSC Preliminary Exam 2020 ) Mark Scored : 0

Q Source: Basics: Human Development Indices

100 Key Biodiversity Areas (KBAs) of IUCN are nationally identified sites of global significance. They
include
1. Important Bird and Biodiversity Areas (IBAs)
2. Alliance for Zero Extinction (AZE) sites
3. Important Sites for Freshwater Biodiversity (ISFB)
4. Important Plant Areas (IPAs)

Select the correct answer using the codes below.


A. 1 and 3 only
B. 2, 3 and 4 only
C. 1 and 4 only
D. 1, 2, 3 and 4

Correct Answer : D

Answer Justification :

Justification: The identification of KBAs is an important approach to address biodiversity


conservation at the site scale i.e. at the level of individual protected areas, concessions and land
management units.

KBAs are identified using globally standardised criteria and thresholds, and have clearly defined
boundaries. There is no maximum or minimum size of sites, because appropriate size varies
according to the socio-economic criteria, such as land use and tenure.

KBAs are seen as an ‘umbrella’ designation, which includes globally important sites for different
taxa and realms as described in the Q Statement.

79
Total Marks : 200
Online Prelims TEST - 29 (TEXTBOOK)
( InsightsIAS Mock Test Series for UPSC Preliminary Exam 2020 ) Mark Scored : 0

Q Source: http://biodiversitya-z.org/content/key-biodiversity-areas-kba

80

You might also like